SlideShare a Scribd company logo
1 of 172
Marc Imhotep Cray, MD
Photo: Color-enhanced scanning electron micrograph of a human spermatocyte fertilizing an oocyte; From: Seeley’s Anatomy & Physiology 10th ed New York, NY: McGraw-Hill 2010
Marc Imhotep Cray, MD
Learning Outcomes cont.
2
Breast:
1. List the inflammatory breast lesions and discuss their etiology.
2. Describe the morphology of the inflammatory breast lesions with
special reference to breast abscess, duct ectasia and fat necrosis.
3. List the proliferative breast lesions and discuss their etiology.
4. Describe the morphology of the proliferative breast lesions with
special reference to fibrocystic changes.
5. Explain the relationship between fibrocystic changes of the breast
and breast cancer.
6. Classify breast tumors
By the end of this sequence the learner will/should be able to:
Marc Imhotep Cray, MD
Female Breast Learning Outcomes cont.
3
7. Describe the morphology of the benign breast tumors with special
reference to fibroadenoma and intraductal papilloma
8. List the risk factors of breast cancer
9. List the histological classification of breast cancer.
10.Explain the prognostic factors of the breast cancer with special
reference to: stage at the time of diagnosis, histological features &
Estrogen-receptor status
11.Describe the gross and microscopic features of different breast
carcinoma with special reference to: in situ ductal carcinoma, in situ
lobular carcinoma, invasive duct carcinoma, invasive lobular
carcinoma & Paget’s disease of the breast.
Marc Imhotep Cray, MD
Learning Outcomes cont.
4
Female genital system:
1. Describe acute and chronic cervicitis.
2. Describe the pathogenesis, risk factors, clinical features and
morphology of squamous cell carcinoma of cervix.
3. Describe the terms used in dysfunctional uterine bleeding and list
its causes.
4. Describe endometritis with special references to various forms of
endometritis.
5. Define endometriosis; explain the theories of endometriosis and
its morphology and complications.
6. Describe endometrial polyps.
FM Reproductive Anatomy_ The Noted Anatomist
Marc Imhotep Cray, MD
FM Genital System Learning Outcomes cont.
5
7. Describe the three types of endometrial hyperplasia.
8. Describe risk factors, etiology, clinical features, morphology and
pathogenesis of endometrial carcinoma as well its morphology.
9. Describe the morphology and clinical features of leiomyomas of the
uterus.
10.List the differences between leiomyoma and leiomyosarcoma of
the uterus.
11.Classify tumors and tumor like lesions of the ovary
12.Describe the pathogenesis and morphology of surface epithelium
tumors, teratomas and granulose theca cell tumors
Marc Imhotep Cray, MD
Female Breast Pathology
6
Marc Imhotep Cray, MD
Female Breast Functional Anatomy
7
 Functional unit of breast is lobule supported by a specialized
intralobular stroma
 Inner luminal epithelial cells produce milk during lactation
 Basally located myoepithelial cells have contractile function to aid
in milk ejection
 Ducts are conduits for milk to reach the nipple
Size of breast is determined primarily by interlobular stroma
increases during puberty and involutes with age
Each normal constituent is a source of both benign and malignant
lesions
Marc Imhotep Cray, MD
Normal breast, gross
8
Klatt EC. Robbins and Cotran Atlas of Pathology, 3rd Ed. 2015.
Marc Imhotep Cray, MD
Breast, mammogram
9
 A mammogram uses a small amount of x-radiation to
visualize the breast parenchyma
 This mammogram shows normal pattern of
lactiferous sinuses and ducts. There is one suspicious
density ( ), however, which could be a carcinoma or
just an area of pronounced sclerosis with fibrocystic
changes
 A mammogram is a useful screening tool to find such
lesions and to determine need for further workup
 A mammogram may detect lesions that are not
palpable
 Women in their 30s begin to have some involution of
lobules and adjacent stroma, and breast tissue
becomes more radiolucent from an increased
composition of adipose tissue replacing fibrous
stroma and lobules
Klatt EC. Robbins and Cotran Atlas of Pathology, 3rd Ed. 2015.
Marc Imhotep Cray, MD 10
Normal breast, microscopic
Klatt EC. Robbins and Cotran Atlas of Pathology, 3rd Ed. 2015.
Marc Imhotep Cray, MD 11
Origins of breast disorders
Kumar, V; Abbas AK (Eds.), Robbins Basic Pathology, 10th Ed. Philadelphia: Elsevier, 2018; Fig.19.22, 736.
 Benign epithelial lesions include
intraductal papillomas that grow in
sinuses below nipple & epithelial
hyperplasia that arises in lobules
 Malignant epithelial lesions are
mainly breast carcinomas, which may
remain in situ or invade into breast
and spread by metastasis
 Specialized intralobular stroma
(green) cells may give rise to
fibroadenomas and phyllodes
tumors, whereas interlobular stroma
(red) may give rise to a variety of
rare benign and malignant tumors
Marc Imhotep Cray, MD
Clinical Presentations of Breast Disease
12
Predominant symptoms and signs of diseases of breast are pain,
inflammatory changes, nipple discharge, “lumpiness,” or a palpable mass
Kumar, V; Abbas AK (Eds.), Robbins Basic Pathology, 10th Ed. Philadelphia: Elsevier, 2018; Fig.19.23, 737.
Marc Imhotep Cray, MD
Risk Factors for Breast Cancer Development
13
Not Modifiable Modifiable
Age Body mass index
BRCA germline mutations Diet
Family history Alcohol
Chest radiation Exogenous estrogen
Race/ethnicity Exercise
Height Smoking
Age at menarche Reproductive history
Age at menopause Age at first full-term delivery
Breast density Lactation
Atypia on prior breast biopsy
Redrawn after Rubin R and Strayer DS Eds. Rubin’s Essential of Pathology:, 6th Ed., 2014, Pg. 536.
Marc Imhotep Cray, MD
Clinical Presentations of Breast Disease:
Key Points
14
 Symptoms affecting breasts are evaluated primarily to
determine if malignancy is present
 Regardless of symptom, underlying cause is benign in majority of
cases
 Breast cancer is most commonly detected by palpation of a
mass in younger women and in unscreened populations and
by mammographic screening in older women
Marc Imhotep Cray, MD
Vignette #1
15
A 30-year-old woman presents to your clinic complaining of bilateral, diffuse
breast pain. Her last menstrual period was 2 weeks ago and she says that she has
felt several painful masses in both breasts during her self-examination in the
shower. She is very concerned because she has a sister-in-law, who was recently
diagnosed with breast cancer. On physical examination, you notice multiple,
palpable masses on both breasts but no changes to the overlying skin. After taking
a detailed history, you learn that the patient does not drink or smoke, has no
immediate family members with a history of breast cancer, and there has been no
nipple discharge. You order a fine needle aspiration cytology, which reveals an
aspirate suggestive of a cyst. You reassure the patient that the lesion is benign and
recommend that she avoid trauma to the affected regions and wear a brassiere
that gives good support and protection.
What is the Diagnosis?
Marc Imhotep Cray, MD
Fibrocystic Changes of the Breast
16
Etiology and Epidemiology: Caused by hormonal imbalance (increased estrogens
and/or decreased progesterones); Peak incidence is between 25 and 50 years old
Pathology:
Four histologic types:
1. Cystic: multiple fluid-filled cysts appear blue (blue-dome cyst) cysts lined by
polygonal cells with eosinophilic granular cytoplasm that are similar to
apocrine epithelium (apocrine metaplasia), may see papillary projections of
cystic epithelium;
2. Epithelial hyperplasia of breast duct: increase in number of epithelial layers
in terminal duct lobules resulting in irregular lumens;
3. Stromal fibrosis: hyperplasia and fibrosis of breast stroma;
4. Sclerosing adenosis: increased number of acini, stromal fibrosis
Marc Imhotep Cray, MD
Fibrocystic Changes of the Breast cont.
17
Clinical Manifestations: Presents with diffuse breast pain (midcycle tenderness)
and multiple, palpable lesions, often bilateral; no changes in overlying skin or
nipple; rapid fluctuation in size of masses is common
Treatment: Symptom management with pain control
NB: Fibrocystic changes are most common breast disorder
 Cystic and stromal fibrosis represent no increased risk for carcinoma, but
 Epithelial hyperplasia and sclerosing adenosis do carry a mildly increased
risk
Marc Imhotep Cray, MD 18
Fibrocystic changes, gross
Klatt EC. Robbins and Cotran Atlas of Pathology, 3rd Ed. 2015.
Marc Imhotep Cray, MD 19
Fibrocystic changes, microscopic
Klatt EC. Robbins and Cotran Atlas of Pathology, 3rd Ed. 2015.
Marc Imhotep Cray, MD 20
Sclerosing adenosis, microscopic
Klatt EC. Robbins and Cotran Atlas of Pathology, 3rd Ed. 2015.
Marc Imhotep Cray, MD 21
Epithelial hyperplasia, microscopic
Klatt EC. Robbins and Cotran Atlas of Pathology, 3rd Ed. 2015.
Marc Imhotep Cray, MD 22
Normal breast, microscopic
Klatt EC. Robbins and Cotran Atlas of Pathology, 3rd Ed. 2015.
Marc Imhotep Cray, MD 23
Normal breast, microscopic
Klatt EC. Robbins and Cotran Atlas of Pathology, 3rd Ed. 2015.
Marc Imhotep Cray, MD 24
Normal breast, microscopic
Klatt EC. Robbins and Cotran Atlas of Pathology, 3rd Ed. 2015.
Marc Imhotep Cray, MD 25
Normal lactating breast, microscopic
Klatt EC. Robbins and Cotran Atlas of Pathology, 3rd Ed. 2015.
Marc Imhotep Cray, MD 26
Acute mastitis, microscopic
Klatt EC. Robbins and Cotran Atlas of Pathology, 3rd Ed. 2015.
Marc Imhotep Cray, MD 27
Breast abscess, gross
Klatt EC. Robbins and Cotran Atlas of Pathology, 3rd Ed. 2015.
Marc Imhotep Cray, MD
Vignette #2
28
A 21-year-old African American woman presents to the clinic
complaining of a large mass in her left breast. She has no immediate
family members with breast cancer. On physical examination,
you notice that the mass is round, rubbery, mobile, and nontender.
It is approximately 4 cm in diameter. You order a needle biopsy that
shows a combination of connective tissue and cystic spaces taking on
a leaflike appearance. You inform the patient that this lesion is most
often benign, but nevertheless you suggest that she be treated with a
local excision to remove the growth.
What is the Diagnosis?
Marc Imhotep Cray, MD
Benign Tumors of the Breast
29
Epidemiology: Fibroadenoma (FA): Occurs in women < 40; tends to occur more
frequently and at a younger age in African American women; Phyllodes tumor
(PT): Occurs most commonly after the age of 50; Intraductal papilloma (IP):
Occurs in middle-aged women
Pathology:
 FA: Gross: small, mobile, rubbery, firm mass with sharp, well-circumscribed
edges; Microscopic: fibroblastic stroma surrounding cystic and glandular
spaces; may regress after menopause and demonstrate calcifications.
 PT: Gross: large, bulky mass of connective tissue and cysts; Microscopic:
cystic spaces on cut section of stroma contains leaflike projections from cyst
walls; leaflike appearance on breast surface; 5%–10% undergoes malignant
change with atypia (cystosarcoma phyllodes).
 IP: Gross: arising from major lactiferous ducts; Microscopic: proliferation of
ductal epithelial tissue in papillary growth manner; apocrine metaplasia
Marc Imhotep Cray, MD
Benign Tumors of the Breast cont.
30
Clinical Manifestations: FA and PT: Increased size and tenderness of
mass with pregnancy or menstrual cycle; no overlying skin changes; no
lymphadenopathy; no nipple retraction
IP: Presents with nipple discharge
Treatment:
FA: No treatment or simple excision
PT: Local excision with wide margin; can recur after resection
IP: Simple excision
Of note: Phyllodes tumor and intraductal papilloma
carry a mildly increased risk of breast carcinoma
Marc Imhotep Cray, MD 31
Fibroadenoma, gross
Klatt EC. Robbins and Cotran Atlas of Pathology, 3rd Ed. 2015.
Marc Imhotep Cray, MD 32
Fibroadenoma, microscopic
Klatt EC. Robbins and Cotran Atlas of Pathology, 3rd Ed. 2015.
Marc Imhotep Cray, MD 33
Phyllodes tumor, microscopic
Klatt EC. Robbins and Cotran Atlas of Pathology, 3rd Ed. 2015.
Marc Imhotep Cray, MD
Phyllodes tumor, mammogram
34
 This mammogram shows a bright, solid
10-cm rounded mass lesion consistent
with a phyllodes tumor
 It still has discrete margins, similar to a
fibroadenoma, but is much larger
 The biologic behavior of a phyllodes
tumor is difficult to predict, and it may
recur locally, but rarely are there high-
grade lesions that can metastasize
 Tend to occur at an older age than do
fibroadenomas, most commonly in sixth
decade
Klatt EC. Robbins and Cotran Atlas of Pathology, 3rd Ed. 2015.
Marc Imhotep Cray, MD 35
Intraductal papilloma, microscopic
Klatt EC. Robbins and Cotran Atlas of Pathology, 3rd Ed. 2015.
Marc Imhotep Cray, MD
Vignette #3
36
A 50-year-old woman presents to your clinic after finding a mass on the upper
outer quadrant of her left breast. After taking a thorough history, you learn that
her mother died from breast cancer and her maternal aunt was also diagnosed
with breast cancer at an early age. The patient started her period at age 11, did
not bear any children, and has not been through menopause. On physical
examination, she is markedly obese and you notice retraction of the skin and the
nipple on her left breast. You locate the mass in question during your breast
examination and find that it is fixed, hard, and nontender. The mass was not
present on her last mammogram dating back 2 years. You also feel palpable
axillary lymph nodes. You schedule the patient for an immediate mammography
and needle biopsy to confirm your suspicions.
What is the Diagnosis?
Marc Imhotep Cray, MD
Breast Carcinoma
37
Etiology and Epidemiology:
Risk factors include:
 Family history of first-degree
relative with breast cancer at
young age (highest risk),
 Autosomal dominant
inheritance of mutations in
BRCA1 or BRCA2 gene,
 female gender,
 Increased age,
 Early first menarche,
 Delayed first pregnancy,
 Nulliparity,
 Late menopause,
 Radiation exposure, and
 exogenous estrogen use
Marc Imhotep Cray, MD
Breast Carcinoma Epidemiology cont.
38
 Breast carcinoma is the most common malignancy of women globally
(excluding nonmelanoma skin cancer) and causes majority of cancer deaths
in women
Incidence in United States decreased slightly in 2002 and then stabilized
 changes attributed to a decrease in use of postmenopausal hormone
therapy and a plateau in number of women undergoing mammographic
screening
 Worldwide incidence and mortality is increasing at an alarming rate
 major factors underlying this trend in developing countries are thought to
be social changes that increase breast cancer risk—specifically, delayed
childbearing, fewer pregnancies, and reduced breastfeeding—combined
with a lack of access to optimal health care
Marc Imhotep Cray, MD
Breast Carcinoma Pathology
39
 Almost all breast malignancies are adenocarcinomas (>95%)
 In the most clinically useful classification system, breast cancers
are divided based on expression of hormone receptors:
 estrogen receptor (ER) and progesterone receptor (PR)—and
 expression of human epidermal growth factor receptor 2 (HER2,
also known as ERBB2 & HER2/neu), into three major groups:
• ER positive (HER2 negative; 50%–65% of cancers)
• HER2 positive (ER positive or negative; 10%–20% of cancers)
• Triple negative (ER, PR, and HER2 negative; 10%–20%
of cancers)
NB: These three groups show striking differences in patient
characteristics, pathologic features, treatment response,
metastatic patterns, time to relapse, and outcome
Marc Imhotep Cray, MD
Age and incidence of breast cancer subtypes
40
 The three hormone receptors
expression groups of the
previous slide show striking
differences in
 patient characteristics,
 pathologic features,
 treatment response,
 metastatic patterns,
 time to relapse, and
 outcome Kumar, V; Abbas AK (Eds.), Robbins Basic Pathology, 10th Ed. Philadelphia:
Elsevier, 2018; Fig.19.26, 740.
Marc Imhotep Cray, MD
FM Breast Anatomy
41
Marc Imhotep Cray, MD
Breast Carcinoma Pathology cont.
42
1. Infiltrating ductal carcinoma: Tumor cells arranged in cords,
islands, or glands embedded in dense fibrous stroma; may arise
from ductal carcinoma in situ (DCIS)
2. Intraductal comedocarcinoma: Sheet of tumor cells confined
within duct; central necrosis; periductal fibrosis with inflammation
3. Inflammatory: Lymphatic involvement of overlying skin
4. Paget disease: Paget cells (large cells w clear halo of pale
cytoplasm) extend from ducts and invade epidermis of nipple;
underlying ductal adenocarcinoma within subareolar excretory
ducts always present
Marc Imhotep Cray, MD
Breast Carcinoma Pathology cont.
43
5. Infiltrating lobular: Often multiple and bilateral; cells line up
(Indian file) with tumor cells surrounding lobule in target fashion;
signet ring cells; may arise from lobular carcinoma in situ (LCIS)
after many years
6. Medullary: Solid sheets of cells with large nucleoli in scant
stroma; lymphocytic infiltrate
7. Mucinous (colloid): Pools of extracellular mucin surrounding
tumor cell clusters; gelatinous consistency
Note: Breast carcinoma is second most
common cause of cancer death among women
Marc Imhotep Cray, MD
Breast Cancer Morphology
44
 Most common location of tumors within breast is in upper
outer quadrant (50%), followed by central portion (20%)
 About 4% of women with breast cancer have bilateral primary
tumors or sequential lesions in the same breast
 Breast cancers are classified morphologically according to
whether they have penetrated the basement membrane
• Those that remain within this boundary are termed in situ
carcinomas, and
• those that have spread beyond it are designated invasive
carcinomas
Marc Imhotep Cray, MD
Breast Cancer Morphology cont.
45
In classification, above main forms of breast carcinoma are as follows:
A. Noninvasive
1. Ductal carcinoma in situ (DCIS)
2. Lobular carcinoma in situ (LCIS)
B. Invasive
1. Invasive ductal carcinoma (includes all carcinomas that are not of a special
type)-70% to 80%
2. Invasive lobular carcinoma- 10% to 15%
3. Carcinoma with medullary features-5%
4. Mucinous carcinoma (colloid carcinoma) -5%
5. Tubular carcinoma-5%
6. Other types
Adapted from: Kumar, V; Abbas AK (Eds.), Robbins Basic Pathology,
10th Ed. Philadelphia, Pa : Elsevier, 2018;742.
Marc Imhotep Cray, MD
Noninvasive (in Situ) Carcinoma
46
 Two morphologic types of noninvasive breast carcinoma: ductal
carcinoma in situ (DCIS) and lobular carcinoma in situ (LCIS)
“ terms ductal and lobular are misleading, as both types of CIS are thought to
arise from cells in terminal duct that give rise to lobules*”
 By definition, both "respect" the basement membrane and do
not invade into stroma or lymphovascular channels
• DCIS has a wide variety of histologic appearances, including solid,
comedo, cribriform, papillary, micropapillary, and "clinging" types
*From: Kumar, V; Abbas AK (Eds.), Robbins Basic Pathology, 10th Ed. Philadelphia: Elsevier, 2018;743.
Marc Imhotep Cray, MD
Carcinoma in situ.
(A) Lobular carcinoma in situ (LCIS).
(B) Ductal carcinoma in situ (DCIS). DCIS partially involves the lobule in the lower half of this photo
and has completely effaced the upper lobules, producing a ductlike appearance.
(C) Mammographic detection of calcifications associated with DCIS.
47
Kumar, V; Abbas AK (Eds.), Robbins Basic Pathology, 10th Ed. Philadelphia: Elsevier, 2018; Fig.19.28,743.
Marc Imhotep Cray, MD
Vignette #4
48
A 49-year-old woman presents to your office concerned about a rash
on her left nipple that has developed over the past month. The rash is
itchy, but painless. On physical examination, you find a large
eczematous-like patch over the left nipple as well as a fixed mass in
the left breast. You perform both a skin biopsy as well as a needle
biopsy of the mass. When the skin biopsy reveals large cells with a
clear halo of pale cytoplasm invading the epidermis, you become
certain that the biopsy of the mass will reveal carcinoma of the breast.
Marc Imhotep Cray, MD
Breast Carcinoma cont.
49
Clinical Manifestations: Painless, usually fixed, hard, nontender mass
often found in upper outer quadrant of breast; retraction of overlying
skin and nipple; palpable axillary lymph nodes
 Infiltrating ductal carcinoma: Firm, fixed, fibrous mass
 Inflammatory: Red, swollen, hot, painful to touch, orange-peel
appearance of skin
 Paget disease: Itchy, scaly, painless, eczematous patches on nipple
 Medullary: Soft, fleshy-consistency mass
Lab findings: Paraneoplastic syndrome with secretion of PTH-related
peptide may lead to hypercalcemia
Imaging: Mammogram with microcalcifications, spiculated or
enlarging mass
Marc Imhotep Cray, MD
Breast Carcinoma cont.
50
Treatment:
 Surgery and radiation therapy;
 Chemotherapy;
 Hormonal therapy (tamoxifen or aromatase inhibitors) for
patients with cancer cells expressing estrogen receptor in their
nuclei (ER/PR+);
 Biologic therapy Trastuzumab (herceptin) for patients with
HER2/neu expression
NB: Metastasis occurs to lymph nodes, lung, liver, and bone.
Marc Imhotep Cray, MD 51
Ductal carcinoma in situ, microscopic
Klatt EC. Robbins and Cotran Atlas of Pathology, 3rd Ed. 2015.
Marc Imhotep Cray, MD 52
Lobular carcinoma in situ, microscopic
Klatt EC. Robbins and Cotran Atlas of Pathology, 3rd Ed. 2015.
Marc Imhotep Cray, MD 53
Paget disease of breast, microscopic
Klatt EC. Robbins and Cotran Atlas of Pathology, 3rd Ed. 2015.
Marc Imhotep Cray, MD 54
Invasive ductal carcinoma, microscopic
Klatt EC. Robbins and Cotran Atlas of Pathology, 3rd Ed. 2015.
Marc Imhotep Cray, MD 55
Invasive ductal
carcinoma, gross
and mammogram
Klatt EC. Robbins and Cotran Atlas of Pathology, 3rd Ed. 2015.
Marc Imhotep Cray, MD 56
Infiltrating ductal carcinoma, gross
and mammogram
Klatt EC. Robbins and Cotran Atlas of Pathology, 3rd Ed. 2015.
Marc Imhotep Cray, MD
Breast Carcinoma Key Points Summary
57
 The lifetime risk of developing breast cancer for an American woman is 1 in 8.
 A majority (75%) of breast cancers are diagnosed after the age of 50.
 The major risk factors for developing breast cancer are related to hormonal
factors and inherited susceptibility.
 About 12% of all breast cancers are caused by identified germline mutations;
BRCAI and BRCA2 genes account for one-half of the cases associated with
single-gene mutations.
 DCIS is a precursor to invasive ductal carcinoma and is most often found on
mammographic screening as calcifications.
 When carcinoma develops in a woman with a previous diagnosis of untreated
DCIS, it is usually is an invasive ductal carcinoma in the same breast.
Marc Imhotep Cray, MD 58
 LCIS is both a marker of increased risk and a precursor lesion.
 When carcinoma develops in a woman with a previous diagnosis of LCIS, two-
thirds are in the same breast and one-third is in the contralateral breast.
 Invasive carcinomas are classified according to histologic type and biologic
type: ER-positive/HER2-negative, HER2-positive, and ER/PR/HER2-negative
(triple-negative). The biologic types of cancer have important differences in
patient characteristics, grade, mutation profile, metastatic pattern, response to
therapy, time to recurrence, and prognosis.
 Prognosis is dependent on the biologic type of tumor, stage, and the availability
of treatment modalities.
Breast Carcinoma Key Points Summary (2)
Marc Imhotep Cray, MD
Ovaries and Uterus Pathology
59
Marc Imhotep Cray, MD
Normal internal genitalia, gross
60
Klatt EC. Robbins and Cotran Atlas of Pathology, 3rd Ed. 2015.
Marc Imhotep Cray, MD
Normal internal genitalia, radiograph
61
Klatt EC. Robbins and Cotran Atlas of Pathology, 3rd Ed. 2015.
Marc Imhotep Cray, MD
Normal fallopian tube, microscopic
62
Klatt EC. Robbins and Cotran Atlas of Pathology, 3rd Ed. 2015.
Marc Imhotep Cray, MD
Normal adult ovary, microscopic
63
Klatt EC. Robbins and Cotran Atlas of Pathology, 3rd Ed. 2015.
Marc Imhotep Cray, MD
Normal ovary, microscopic
64
Klatt EC. Robbins and Cotran Atlas of Pathology, 3rd Ed. 2015.
Marc Imhotep Cray, MD
Corpus luteum, gross
65
Klatt EC. Robbins and Cotran Atlas of Pathology, 3rd Ed. 2015.
Marc Imhotep Cray, MD
Menstrual cycle capsular summary:
 Menstrual cycle is divided into follicular phase and luteal phase
Ovulation defines transition between these two phases
 During follicular phase, gonadotroph cells of anterior pituitary gland
secrete LH and FSH in response to pulsatile GnRH stimulation
 Circulating LH and FSH promote growth and maturation of ovarian follicles
 Developing follicles secrete increasing amounts of estrogen
 At first, estrogen has an inhibitory effect on gonadotropin release Just
before midpoint in menstrual cycle, however, estrogen exerts a brief
positive feedback effect on LH (LH Surge) and FSH release This is
followed by follicular rupture and release of an egg into fallopian tube
 During second half of cycle, corpus luteum secretes both estrogen and
progesterone
 Progesterone induces a change in endometrium from a proliferative to a
secretory type
 If fertilization and implantation of a blastocyst do not occur within 14 days
after ovulation corpus luteum involutes secretion of estrogen and
progesterone declines, menses occurs, and a new cycle begins
Cairo CW, Simon JB, Golan DE. (Eds.). Principles of Pharmacology:
The Pathophysiologic Basis of Drug Therapy. LLW, 2012.
Marc Imhotep Cray, MD
Hormonal Control of the Menstrual Cycle, Animation.
67
Online Version
Marc Imhotep Cray, MD
Menstruation terminology
68
 Dysmenorrhea= Pain with menses often associated with
endometriosis
 Oligomenorrhea > 35-day cycle
 Polymenorrhea < 21-day cycle
 Metrorrhagia= Frequent or irregular menstruation
 Menorrhagia= Heavy menstrual bleeding > 80 mL blood loss or > 7
days of menses
 Menometrorrhagia= Heavy, irregular menstruation
Marc Imhotep Cray, MD
Vignette #5
69
A 45-year-old woman presents to the clinic complaining of vague
abdominal pain for the past 4 days. She describes her pain as more of
a pelvic pressure that is generalized bilaterally. Her last menstrual
period was 2 weeks ago and she states that she has regular menstrual
cycles. She denies the possibility of pregnancy and is currently not
taking oral contraceptives. She has not had any changes in digestive
functions and denies any nausea, vomiting, constipation, or diarrhea.
She has no family history of ovarian cancer. When an ultrasound
confirms your suspicions, you place the patient on oral contraceptive
pills, believing that her symptoms will disappear in 2 months, and you
schedule her for a follow-up ultrasound.
What is the Diagnosis?
Marc Imhotep Cray, MD
Ovarian Cysts
70
Etiology and Epidemiology:
 Follicular (F) cyst: Associated with hyperestrinism and endometrial
hyperplasia; most common cause of ovarian enlargement; mostly found
during menstrual years
 Corpus luteum (CL) cyst: Found during menstrual years
 Theca-lutein (TL) cyst: Associated with choriocarcinoma, hydatidiform
moles, and clomiphene (synthetic gonadotropin) therapy
Pathology
 F: Often bilateral; distention of unruptured Graafian follicle; lined by
granulosa cells
 CL: Often unilateral; contains clear fluid; lined by yellowish luteal cells with
cytoplasmic lipid droplets; may hemorrhage into persistent mature corpus
luteum
 TL: Often bilateral and multiple; lined by luteinized theca cells
Marc Imhotep Cray, MD
Ovarian Cysts cont.
71
Clinical Manifestations: All may be asymptomatic or present with
pelvic pressure/pain or vague GI discomfort
 F: Pain not associated with menstruation
 CL: Delayed menstruation
 TL: Amenorrhea
Lab findings:
hCG elevated as a result of trophoblastic proliferation
Treatment:
 F: Often disappears with 2-month regimen of oral
contraceptives; follow with serial ultrasounds; laparoscopic
removal if persistent
 CL and TL: Cyst removal or unilateral oophorectomy
Marc Imhotep Cray, MD
Vignette #6
72
A 30-year-old white woman presents to the fertility clinic with her husband,
complaining of the inability to conceive. The couple has already checked the
husband’s sperm motility and count, which are within the normal range. The
patient informs you that she has not menstruated for the last 4 months and that
she has had a very irregular and sporadic menstrual cycle all of her life. You
perform a physical examination, finding that the patient is obese with an
inordinate amount of facial hair. She denies any abnormal uterine bleeding. You
order serum studies that show elevated plasma LH and testosterone and
decreased FSH levels. Based on these findings, you inform the patient that
weight reduction will be the most effective treatment for restoring ovulation
and that adjunct therapy with clomiphene can aid in ovulation.
What is the Diagnosis?
Marc Imhotep Cray, MD
Polycystic Ovarian Syndrome (Stein-Leventhal
Syndrome)
73
Etiology and Epidemiology: Etiology unclear, but it is believed that a dysregulation
of enzymes involved in androgen biosynthesis may be caused by increased LH
secretion thereby resulting in excessive production of androgens; associated
with obesity, Cushing syndrome, congenital adrenal hyperplasia, genetic
predisposition, and androgen-secreting adrenal tumors
Common endocrine disorder affecting 2%–5% of women during reproductive age
Pathology and Pathophysiology:
Pathophysiology: Increased androgen production causes anovulation, multiple
follicular cysts, and theca cell hyperplasia
Gross: Ovaries enlarged; pearly white thickened ovarian capsule; multiple cysts
Microscopic: Cysts have granulosa cell layer and luteinized theca cells; cortical
stromal fibrosis
Marc Imhotep Cray, MD
PCOS cont.
74
Clinical Manifestations: Amenorrhea; infertility; obesity; hirsutism (in
70%); insulin resistance with increased risk of diabetes; virilism;
increased risk of breast and endometrial carcinoma
Lab findings: Increased LH, decreased FSH, increased testosterone,
evidence of insulin resistance
Treatment: Weight loss; oral contraceptives; metformin for insulin
resistance; gonadotropin analogs; ovulation induction with clomiphene
Marc Imhotep Cray, MD
Polycystic ovary, MRI
75
Klatt EC. Robbins and Cotran Atlas of Pathology, 3rd Ed. 2015.
Marc Imhotep Cray, MD
Polycystic ovarian syndrome, microscopic
76
Klatt EC. Robbins and Cotran Atlas of Pathology, 3rd Ed. 2015.
Marc Imhotep Cray, MD
Vignette #7
77
A 40-year-old woman presents to the emergency room with
generalized abdominal pain and pelvic pressure. After taking a more
complete history, you learn that she has not passed stool for the last
3 days and vomited this morning. She tells you that the pain has
been steady over the last week. You order an abdominal/pelvic CT
scan, which is inconclusive, but does demonstrate bilateral
enlargement of the ovaries. The patient is taken to the operating
room for an exploratory laparotomy that shows extensive mucinous
ascites, cystic epithelial implants on the peritoneal surfaces, and
several adhesions. You believe that the patient’s mucinous peritoneal
involvement is caused by her ovarian condition.
What is the Diagnosis?
Marc Imhotep Cray, MD
Ovarian Tumors of Surface Epithelium Origin
78
Epidemiology: Usually occurs in women > 20 years of age
Pathology:
 Serous cystadenoma: Bilateral, benign cyst with fallopian tube-like epithelium
 Papillary serous cystadenocarcinoma: Bilateral, malignant cysts lined by
stratified atypical epithelium; papillary growth; psammoma bodies
 Mucinous cystadenoma: Multilocular, benign cysts with columnar cells filled
with mucin
 Mucinous cystadenocarcinoma: Malignant tumor with mucus-secreting
atypical columnar epithelium; loss of gland architecture; necrosis
 Brenner tumor: Benign tumor with nests of cells resembling bladder
transitional epithelium interspersed in fibrous stroma
 Endometrioid tumor: Malignant tumor resembling endometrium
Clear cell tumor: Rare, malignant tumor composed of sheets of clear cells
Marc Imhotep Cray, MD
Ovarian Tumors of Surface Epithelium
Origin cont.
79
Clinical Manifestations: Mild, nonspecific abdominal discomfort;
malignant forms may present with weakness, weight loss, and
anorexia; pseudomyxoma peritonei (intraperitoneal accumulation of
mucinous material) is associated with mucinous cystadenocarcinoma
Lab finding: Elevated CA-125 in ovarian carcinomas
Treatment: Tumor removal; oophorectomy or hysterectomy;
chemotherapy
Of note: Ovarian epithelial tumors make up 75% of ovarian tumors.
Marc Imhotep Cray, MD
Serous cystadenoma, gross
80
Klatt EC. Robbins and Cotran Atlas of Pathology, 3rd Ed. 2015.
Marc Imhotep Cray, MD 81
Serous
cystadenoma,
MRI and CT
image
Klatt EC. Robbins and Cotran Atlas of Pathology, 3rd Ed. 2015.
Marc Imhotep Cray, MD
Multiloculated ovarian tumor, gross
82
Klatt EC. Robbins and Cotran Atlas of Pathology, 3rd Ed. 2015.
Marc Imhotep Cray, MD
Cystadenoma, serous, mucinous, microscopic
83
Klatt EC. Robbins and Cotran Atlas of Pathology, 3rd Ed. 2015.
Marc Imhotep Cray, MD
Borderline serous tumor, microscopic
84
Klatt EC. Robbins and Cotran Atlas of Pathology, 3rd Ed. 2015.
Marc Imhotep Cray, MD
Borderline tumor, gross
85
Klatt EC. Robbins and Cotran Atlas of Pathology, 3rd Ed. 2015.
Marc Imhotep Cray, MD
Cystadenocarcinoma & peritoneal
metastases, CT images
86
Klatt EC. Robbins and Cotran Atlas of Pathology, 3rd Ed. 2015.
Marc Imhotep Cray, MD
Cystadenocarcinoma, gross
87
Klatt EC. Robbins and Cotran Atlas of Pathology, 3rd Ed. 2015.
Marc Imhotep Cray, MD
Cystadenocarcinoma, microscopic
88
Klatt EC. Robbins and Cotran Atlas of Pathology, 3rd Ed. 2015.
Marc Imhotep Cray, MD
Endometrioid tumor, microscopic
89
Klatt EC. Robbins and Cotran Atlas of Pathology, 3rd Ed. 2015.
Marc Imhotep Cray, MD
Brenner tumor, microscopic
90
Klatt EC. Robbins and Cotran Atlas of Pathology, 3rd Ed. 2015.
Marc Imhotep Cray, MD
Vignette #8
91
An 18-year-old white woman is admitted to your gynecologic service with a 1-
month history of increasing abdominal pain and a rapidly enlarging pelvic
mass. While admitting the patient, you perform a bimanual pelvic examination
that reveals a left ovarian mass. This finding is consistent with the admitting
note and was confirmed by abdominal/pelvic CT, which showed that the left
ovary was twice as large as the right ovary. Serum studies demonstrate an
elevated AFP level. There is no inguinal lymph node involvement and no signs
of metastasis on imaging studies. Based on your findings, you perform a
unilateral oophorectomy and send the diseased ovary for pathologic analysis.
Histology shows glomerulus-like structures composed of a central blood vessel
enveloped by germ cells within a space similarly lined by germ cells. These
histologic bodies confirm your suspected diagnosis and you start the patient
on combination chemotherapy
What is the Diagnosis?
Marc Imhotep Cray, MD
Ovarian Tumors of Germ Cell Origin
92
Etiology and Epidemiology: Risk factors include nulliparity, positive family history
of ovarian cancer, mutations in BRCA1 and BRCA2 genes and high expression of
HER2/neu oncogene; Occurs most commonly in children and young adults (except
for teratomas, which can occur at all ages)
Pathology:
 Dysgerminoma: Malignant unilateral tumor comprised of large vesicular cells
with clear cytoplasm and central nuclei; analogous to male testicular seminoma
 Yolk sac tumor: Malignant tumor with Schiller-Duval bodies (glomerulus-like
structure composed of central blood vessel enveloped by germ cells)
 Choriocarcinoma: Aggressive and malignant tumor with areas of necrosis and
hemorrhage composed of neoplastic syncytiotrophoblasts and cytotrophoblasts
 Teratoma: 90% of germ cell tumors; mature teratomas (dermoid cysts) are
benign; immature are malignant ; Histology: structures from all germ layers.
 Struma ovarii: Unilateral ovarian teratoma composed of thyroid tissue
Marc Imhotep Cray, MD
Ovarian Tumors of Germ Cell Origin cont.
93
Clinical Manifestations: Mild, nonspecific abdominal discomfort;
struma ovarii may lead to hyperthyroidism
Lab findings: Increased AFP (yolk sac), increased hCG (choriocarcinoma)
Tx: Tumor removal; oophorectomy or hysterectomy; chemotherapy
Note: Germ cell tumors make up 25% of ovarian tumors
Marc Imhotep Cray, MD 94
Mature cystic teratoma, gross
Klatt EC. Robbins and Cotran Atlas of Pathology, 3rd Ed. 2015.
Marc Imhotep Cray, MD 95
Mature cystic teratoma, CT image
Klatt EC. Robbins and Cotran Atlas of Pathology, 3rd Ed. 2015.
Marc Imhotep Cray, MD 96
Mature cystic teratoma, microscopic
Klatt EC. Robbins and Cotran Atlas of Pathology, 3rd Ed. 2015.
Marc Imhotep Cray, MD 97
Dysgerminoma, gross
Klatt EC. Robbins and Cotran Atlas of Pathology, 3rd Ed. 2015.
Marc Imhotep Cray, MD 98
Dysgerminoma, microscopic
Klatt EC. Robbins and Cotran Atlas of Pathology, 3rd Ed. 2015.
Marc Imhotep Cray, MD
Vignette #9
99
A 12-year-old girl is brought into the emergency room because of
heavy vaginal bleeding. The child is also complaining of GI discomfort
and pelvic pressure. She states that she had her last menstruation 2
weeks earlier and she is not sexually active. Her first menstruation was
at the age of 10. On physical examination, the child appears to have
undergone precocious puberty. A pelvic examination reveals a
palpable right ovarian mass, which is confirmed by a CT scan. Serum
studies show a significantly increased level of circulating estrogen.
When a biopsy reveals distinctive, gland-like structures filled with an
acidophilic material, you worry that this patient may have an
increased risk of developing endometrial carcinoma later in life if this
mass is not removed.
What is the Diagnosis?
Marc Imhotep Cray, MD
Ovarian Tumors of Sex Cord–Stromal Origin
100
Etiology and Epidemiology: Risk factors include nulliparity, positive family
history of ovarian cancer, mutations in BRCA1 and BRCA2 genes, and high
expression of HER2/neu oncogene; Affects all age groups
Pathology and Pathophysiology:
 Ovarian fibroma-thecoma (OFT): Secretes estrogen; tumor composed of
round lipid-containing cells in addition to well-differentiated fibroblasts
 Granulosa cell tumor (GCT): Secretes estrogen causing endometrial
hyperplasia/carcinoma in adults; characterized by Call-Exner bodies (small
follicles filled with eosinophilic secretions) and small cuboidal, deeply
stained granulosa cells arranged in anastomotic cords
 Sertoli-Leydig cell tumor (SLCT): Secretes androgens; tumor composed of
Sertoli or Leydig cells interspersed with stroma
Marc Imhotep Cray, MD
Ovarian Tumors of Sex Cord–Stromal Origin cont.
101
Clinical Manifestations: Mild, nonspecific abdominal discomfort
 OFT: Presents with Meig syndrome (triad of ovarian tumor,
ascites, and hydrothorax)
 GCT: Vaginal bleeding from endometrial hyperplasia; cystic
disease of breast; endometrial carcinoma
 SLCT: Virilism
Lab findings: Increased estrogen levels (OFT and GCT)
Treatment: Tumor removal; oophorectomy or hysterectomy;
chemotherapy
Note: Ovarian sex cord–stromal tumors are rare
Marc Imhotep Cray, MD 102
Granulosa–theca cell tumor, gross
Klatt EC. Robbins and Cotran Atlas of Pathology, 3rd Ed. 2015.
Marc Imhotep Cray, MD 103
Granulosa cell tumor, microscopic
Klatt EC. Robbins and Cotran Atlas of Pathology, 3rd Ed. 2015.
Marc Imhotep Cray, MD 104
Thecoma-fibroma, gross
Klatt EC. Robbins and Cotran Atlas of Pathology, 3rd Ed. 2015.
Marc Imhotep Cray, MD 105
Thecoma-fibroma, microscopic
Klatt EC. Robbins and Cotran Atlas of Pathology, 3rd Ed. 2015.
Marc Imhotep Cray, MD
Vignette #10
106
A 31-year-old woman presents to the clinic complaining of abnormal
vaginal bleeding. She states that her last menstrual period was 1
week ago and that she has not experienced abnormal bleeding
before. Her past medical history is notable for two prior episodes of
pelvic inflammatory disease in the last year. She has also been trying
to conceive for the last year without success. On pelvic examination,
redness and inflammation of the cervix is visible, but no discharge is
apparent. You obtain an endometrial biopsy, which you suspect will
show plasma cells along with macrophages and leukocytes in the
glandular lumen. You begin empiric antibiotic therapy to prevent
possible sequelae from the suspected diagnosis.
What is the Diagnosis?
Marc Imhotep Cray, MD
Endometritis
107
Etiology:
 Acute endometritis: Caused by trauma, Staphylococcus aureus and
Streptococcus species; usually occurs after delivery or miscarriage;
 Chronic endometritis: Caused by granulomatous disease, chronic PID,
postpartal or postabortal states, and TB
Pathology:
Endometrium: Plasma cells seen with macrophages and lymphocytes
Clinical Manifestations:
Acute: Presents with inflammation after delivery or miscarriage
Chronic: Presents with abnormal vaginal bleeding, pain, discharge, and infertility
Treatment: Antibiotic Tx (helps to prevent other sequelae, such as salpingitis)
Of Note: Endometrial polyps are masses composed of endometrial tissue within
endometrial cavity  occur in women older than 40 years of age and may result in
uterine bleeding, but are usually benign
Marc Imhotep Cray, MD 108
Acute endometritis, microscopic
Klatt EC. Robbins and Cotran Atlas of Pathology, 3rd Ed. 2015.
Marc Imhotep Cray, MD 109
Chronic endometritis, microscopic
Klatt EC. Robbins and Cotran Atlas of Pathology, 3rd Ed. 2015.
Marc Imhotep Cray, MD 110
Granulomatous endometritis, microscopic
Klatt EC. Robbins and Cotran Atlas of Pathology, 3rd Ed. 2015.
Marc Imhotep Cray, MD
Vignette #11
111
A 24-year-old woman presents to the clinic complaining of increased
pain and bleeding during menstruation. Her last three
menstruations have been accompanied by increasing intensity of
cramping and larger amounts of blood. She tells you that her
menstrual cycle has been irregular for the last 6 months. After
taking a complete history, you learn that she has been having
increased pelvic pain with intercourse. On pelvic examination, you
palpate fixed, bilateral ovarian masses and an MRI reveals chocolate
cysts on the ovary. You begin the patient on oral contraceptives and
you suggest surgical removal of the masses if the pain persists.
What is the Diagnosis?
Marc Imhotep Cray, MD
Endometriosis
112
Etiology and Epidemiology: Etiology unknown, but thought to be
caused by a combination of genetic, hormonal, and immune factors;
Afflicts 10% of women, usually between the ages of 20 and 30
Pathology:
 Gross: Non-neoplastic nodules, composed of endometrial tissue in
abnormal locations outside the uterus, including ovary (most
common), uterine ligaments, rectovaginal septum, and pelvic
peritoneum; presence of chocolate cysts on ovaries that have
resulted from cyclic bleeding (menstrual type) of ectopic
endometrial tissue
 Microscopic: Endometrial glands; endometrial stroma; presence of
hemosiderin pigment
Marc Imhotep Cray, MD
Endometriosis cont.
113
Clinical Manifestations: Presents with fixed, palpable, bilateral
ovarian masses, pain during menses (dysmenorrhea), and pain during
intercourse (dyspareunia); menstrual irregularities may lead to
infertility presenting complaint of 30%–40% of pts.
Treatment: Oral contraceptives; progesterone; danazol; GnRH;
surgical removal/coagulation (cauterization) of lesion
Note: Adenomyosis is endometriosis within the myometrium and
usually presents with uterine enlargement and irregular bleeding
Marc Imhotep Cray, MD 114
Adenomyosis, MRI
Klatt EC. Robbins and Cotran Atlas of Pathology, 3rd Ed. 2015.
Marc Imhotep Cray, MD 115
Adenomyosis, gross
Klatt EC. Robbins and Cotran Atlas of Pathology, 3rd Ed. 2015.
Marc Imhotep Cray, MD 116
Adenomyosis, microscopic
Klatt EC. Robbins and Cotran Atlas of Pathology, 3rd Ed. 2015.
Marc Imhotep Cray, MD 117
Endometriosis, gross
Klatt EC. Robbins and Cotran Atlas of Pathology, 3rd Ed. 2015.
Marc Imhotep Cray, MD 118
Endometriosis, gross
Marc Imhotep Cray, MD 119
Endometriosis, microscopic
Klatt EC. Robbins and Cotran Atlas of Pathology, 3rd Ed. 2015.
Marc Imhotep Cray, MD 120
Endometrial polyp, gross
Klatt EC. Robbins and Cotran Atlas of Pathology, 3rd Ed. 2015.
Marc Imhotep Cray, MD
Vignette #12
121
A 35-year-old African American woman presents to the clinic
complaining of more frequent menstrual periods and profuse
menstruation. She also complains of increased weakness and fatigue
during her menstruations. On physical examination, you find a firm
abdominal mass in the pelvic region. When questioned, she states
that she was aware of the mass, but she notes that the mass often
only appears during menstrual periods. When a biopsy of the mass
reveals whorled bundles of smooth muscle cells, you reassure this
patient that her condition is unlikely to proceed to malignancy.
What is the Diagnosis?
Marc Imhotep Cray, MD
Leiomyoma (fibroids)
122
Etiology and Epidemiology: Etiology unknown, but chromosomal abnormalities
have been found in tumor cells; Most common benign neoplasm of female
genital tract; Most commonly seen in women of reproductive age with an
increased incidence in African Americans
Pathology:
Gross: Multiple, enlarged, irregular, heterogenous tumors in the myometrium
(intramural), beneath endometrium (submucosal), or beneath serosa
(subserosal); tumor is estrogen-sensitive with its size increasing during pregnancy
and decreasing with menopause
Microscopic: Whorled pattern of smooth muscle bundles; rare mitoses in muscle
cells, although cellular atypia and giant cells may be present
Marc Imhotep Cray, MD
Leiomyoma (fibroids) cont.
123
Clinical Manifestations: Profuse menstruation; frequent menstrual periods;
acute or recurrent pelvic pain; urinary frequency (owing to compression of
bladder); infertility
Lab findings: Iron deficiency anemia (owing to blood loss)
Treatment: Myomectomy or hysterectomy; uterine artery embolization
Note: Also called uterine fibroids, leiomyomas do not progress to
leiomyosarcomas
 Leiomyosarcomas are fleshy, irregular tumors with areas of necrosis and
hemorrhage that arise de novo and may protrude from cervix
 Leiomyosarcomas are more prevalent among African Americans, are
malignant, and can be treated with combination chemotherapy
Marc Imhotep Cray, MD 124
Leiomyomata, gross
Klatt EC. Robbins and Cotran Atlas of Pathology, 3rd Ed. 2015.
Marc Imhotep Cray, MD
Uterine fibroids as seen during laparoscopic
surgery
125
https://commons.wikimedia.org/wiki/File:Uterine_fibroids.jpg
Marc Imhotep Cray, MD 126
Leiomyomata, MRI and CT image
Klatt EC. Robbins and Cotran Atlas of Pathology, 3rd Ed. 2015.
Marc Imhotep Cray, MD 127
Leiomyoma, microscopic
Klatt EC. Robbins and Cotran Atlas of Pathology, 3rd Ed. 2015.
Marc Imhotep Cray, MD 128
Leiomyosarcoma, gross
Klatt EC. Robbins and Cotran Atlas of Pathology, 3rd Ed. 2015.
Marc Imhotep Cray, MD 129
Leiomyosarcoma, microscopic
Klatt EC. Robbins and Cotran Atlas of Pathology, 3rd Ed. 2015.
Marc Imhotep Cray, MD
Vignette #13
130
A 60-year-old woman presents to the clinic with postmenopausal
vaginal bleeding. After taking a complete history, you learn that she is
nulliparous and suffers from type 2 diabetes, which is well-controlled
with diet and insulin. On physical examination, the woman is obese
and has a blood pressure reading of 150/96. You decide to perform a
PAP smear as well as an endometrial biopsy. Based on the patient’s
presenting signs and medical history, you are worried that you might
find well-defined gland patterns lined by malignant stratified
columnar epithelial cells on endometrial biopsy.
What is the Diagnosis?
Marc Imhotep Cray, MD
Endometrial Carcinoma
131
Etiology and Epidemiology: Risk factors include unopposed estrogen use,
obesity, diabetes, HTN, nulliparity, and late menopause
Peak incidence is between 55 and 65 years of age
Endometrial carcinoma is most common gynecologic malignancy
Pathology: Typically preceded by endometrial hyperplasia
Gross: Localized polypoid tumor or diffuse tumor involving entire endometrial
surface
Microscopic: Adenocarcinoma characterized by well-defined gland patterns lined
by malignant stratified columnar epithelial cells; may see some squamous cells
Marc Imhotep Cray, MD
Endometrial Ca. cont.
132
Clinical Manifestations: Presents with postmenopausal vaginal bleeding,
leading to early diagnosis; may cause obstruction of cervix with collection of pus
(pyometra) or blood (hematometra) presenting with lower abdominal pain
Treatment: Total hysterectomy and bilateral salpingo-oophorectomy; radiation
therapy
Important Note:
 Endometrial hyperplasia is abnormal endometrial gland proliferation caused
by excess estrogen (eg, polycystic ovarian syndrome, estrogen-secreting
ovarian tumor, estrogen replacement therapy)
 It manifests clinically with postmenopausal vaginal bleeding  may lead to
endometrial cancer depending on degree of atypia
Marc Imhotep Cray, MD 133
Endometrial atypical hyperplasia, microscopic
Klatt EC. Robbins and Cotran Atlas of Pathology, 3rd Ed. 2015.
Marc Imhotep Cray, MD 134
Endometrial carcinoma, gross
Klatt EC. Robbins and Cotran Atlas of Pathology, 3rd Ed. 2015.
Marc Imhotep Cray, MD 135
Endometrial carcinoma, gross (2)
Klatt EC. Robbins and Cotran Atlas of Pathology, 3rd Ed. 2015.
Marc Imhotep Cray, MD 136
Endometrial carcinoma, type I, microscopic
Klatt EC. Robbins and Cotran Atlas of Pathology, 3rd Ed. 2015.
Marc Imhotep Cray, MD 137
Endometrial carcinoma, type II, microscopic
Klatt EC. Robbins and Cotran Atlas of Pathology, 3rd Ed. 2015.
Marc Imhotep Cray, MD
Vignette #14
138
A 42-year-old woman presents to the clinic complaining of postcoital
bleeding and vaginal discharge. She complains of a 3-month history of
spotting in her underwear after intercourse and an odorous vaginal
discharge that is not purulent. Her social history is significant for past
practice of prostitution and her past medical history is significant for
several STDs that were appropriately treated. She has not had a
routine PAP smear in over 10 years. After a PAP smear reveals
abnormal cells, you perform a cervical biopsy, worrying that you may
find invasive malignant cells in the cervix and adjacent koilocytosis.
You fear that the patient will need to undergo a hysterectomy with
possible adjunct radiation therapy if your suspected diagnosis is
confirmed.
What is the Diagnosis?
Marc Imhotep Cray, MD
Dysplasia, Carcinoma in Situ, and Squamous
Cell Carcinoma of the Cervix
139
Etiology and Epidemiology: Associated with human papilloma virus (HPV) types
16, 18, 31, and 33, early age of first intercourse and multiple sexual partners
Occurs most commonly between the ages of 30 and 45
Pathology: Cervical dysplasia: Involves squamocolumnar junction; characterized
by cells with hyperchromatic nuclei, irregular nuclear contours, and scant
cytoplasm; epithelial growth begins at basal layer extending outward; classified as
Cervical Intraepithelial Neoplasia (CIN) Subtypes Grades I-III (or Squamous
Intraepithelial Lesions Low-grade to High-grade )
 CIN I (LSIL) is characterized by atypical undifferentiated cells only in lower third
of epithelium, whereas
 CIN II (HSIL) superficial layer of cells still shows differentiation and koilocytosis
 CIN III (HSIL) has atypia through > 2/3 thickness of epithelium, koilocytotic
change usually is absent
Marc Imhotep Cray, MD
Dysplasia, Carcinoma in Situ, and Squamous
Cell Carcinoma of the Cervix (2)
140
Pathology cont:
Cervical carcinoma in situ (CIS): Dysplastic cells extending through entire
epithelium, but without invasion of basement membrane
Invasive cervical carcinoma (ICC):
Gross: can be exophytic, ulcerating, or infiltrating mass
Microscopic: usually squamous cell carcinoma with large cells and keratinization;
can be adenocarcinoma or undifferentiated carcinoma; arises from preexisting
CIN at squamocolumnar junction; non-neoplastic epithelial cells often
demonstrate koilocytosis (assoc. w. HPV infection)
Koilocytes, also known as halo cells, are a type of epithelial cell that develops
following a human papillomavirus (HPV) infection; They are structurally different
from other epithelial cells, in that, their nuclei are an irregular size, shape, or color
Marc Imhotep Cray, MD
Natural History of Squamous Intraepithelial
Lesions (SILs)
141
Lesion Regress Persist Progress
LSIL (CIN I) 60% 30% 10% (to HSIL)
HSIL (CINII,III) 30% 60% 10% (to carcinoma)*
*Progression within 10 years. LSIL, Low-grade SIL; HSIL, high-grade SIL
Redrawn after Kumar V and Abbas AK. Robbins and Cotran Basis Pathology, 10th Ed. Philadelphia: Saunders, 2018, Fig. 19.1 , Pg. 718.
Remember:
Cervical Intraepithelial Neoplasia (CIN) Subtypes Grades I-III (or Squamous Intraepithelial Lesions
Low-grade to High-grade )
 CIN I (LSIL) is characterized by atypical undifferentiated cells only in lower third of epithelium,
whereas,
 CIN II (HSIL) superficial layer of cells still shows differentiation and koilocytosis
 CIN III (HSIL) has atypia through > 2/3 thickness of epithelium, koilocytotic change usually is absent
Marc Imhotep Cray, MD 142
Three Stages of squamous intraepithelial lesions (SIL)
Spectrum of squamous intraepithelial lesions (SIL) with normal squamous epithelium for
comparison: LSIL with koilocytotic atypia; HSIL w progressive atypia in all layers of the
epithelium; and HSIL w diffuse atypia and loss of maturation (carcinoma in situ, far right image)
Kumar V and Abbas AK. Robbins and Cotran Basis Pathology, 10th Ed. Philadelphia: Saunders, 2018, Fig. 19.6 , Pg. 719.
Marc Imhotep Cray, MD 143
Cytologic features of squamous intraepithelial
lesion (SIL) in a Papanicolaou Smear
Superficial squamous cells may stain either red or blue.
(A) Normal exfoliated superficial squamous epithelial cells. (B) Low-grade squamous intraepithelial lesion (LSIL).
(C and D) Both high-grade squamous intraepithelial lesions (HSILs). Note the reduction in cytoplasm and the increase in
the nucleus-to-cytoplasm ratio as the grade of the lesion increases. This observation reflects progressive loss of cellular
differentiation on surface of cervical lesions from which these cells are exfoliated
Kumar V and Abbas AK. Robbins and Cotran Basis Pathology, 10th Ed. Philadelphia: Saunders, 2018, Fig. 19.7 , Pg. 719.
Marc Imhotep Cray, MD
Dysplasia, Carcinoma in Situ, and Squamous Cell
Carcinoma of Cervix (3)
144
Clinical Manifestations: ICC: Irregular vaginal bleeding; postcoital
spotting; cervical ulceration; nonpurulent discharge; dysuria; invasive
disease can obstruct ureters and lead to renal failure
Treatment ICC: Hysterectomy; radiation therapy; prevention with
HPV vaccine
NB: Screening for cervical cancer with PAP smears should begin 3
years after intercourse and no later than age 21 and continue every
1–3 years until age 70
Marc Imhotep Cray, MD 145
Normal cervix, gross
Klatt EC. Robbins and Cotran Atlas of Pathology, 3rd Ed. 2015.
Marc Imhotep Cray, MD 146
Normal cervix, microscopic
Klatt EC. Robbins and Cotran Atlas of Pathology, 3rd Ed. 2015.
Marc Imhotep Cray, MD 147
Normal cervical transformation zone, microscopic
Klatt EC. Robbins and Cotran Atlas of Pathology, 3rd Ed. 2015.
Marc Imhotep Cray, MD 148
Chronic cervicitis, gross
Klatt EC. Robbins and Cotran Atlas of Pathology, 3rd Ed. 2015.
Marc Imhotep Cray, MD 149
Chronic cervicitis, microscopic
Klatt EC. Robbins and Cotran Atlas of Pathology, 3rd Ed. 2015.
Marc Imhotep Cray, MD 150
Cervical squamous metaplasia, microscopic
Klatt EC. Robbins and Cotran Atlas of Pathology, 3rd Ed. 2015.
Marc Imhotep Cray, MD 151
Human papillomavirus effect, microscopic
Klatt EC. Robbins and Cotran Atlas of Pathology, 3rd Ed. 2015.
Marc Imhotep Cray, MD 152
Cervical squamous dysplasia, Pap smear
Klatt EC. Robbins and Cotran Atlas of Pathology, 3rd Ed. 2015.
Marc Imhotep Cray, MD 153
Cervical squamous carcinoma, Pap smear
Klatt EC. Robbins and Cotran Atlas of Pathology, 3rd Ed. 2015.
Marc Imhotep Cray, MD 154
Squamous cell carcinoma, gross
Klatt EC. Robbins and Cotran Atlas of Pathology, 3rd Ed. 2015.
Marc Imhotep Cray, MD 155
Squamous cell carcinoma, gross (2)
Klatt EC. Robbins and Cotran Atlas of Pathology, 3rd Ed. 2015.
Marc Imhotep Cray, MD 156
Squamous cell carcinoma, gross (3)
Klatt EC. Robbins and Cotran Atlas of Pathology, 3rd Ed. 2015.
Marc Imhotep Cray, MD
Vignette #15
157
A 27-year-old woman presents to the emergency room with lower
abdominal pain, chills, fever, and a purulent vaginal discharge. After
taking a history, you learn that she has had multiple sexual partners
over the last year and has not practiced safe sex. She states that the
pain started 3 days ago with subsequent fevers, chills, and night
sweats. On physical examination, she is febrile with a temperature of
102.4°F. Pelvic examination is significant for cervical motion
tenderness with foul-smelling, purulent, cervical discharge. Her
pregnancy test is negative. You admit the patient to the hospital and
you begin her on IV cefoxitin plus doxycycline while awaiting the
results of her endocervical culture.
What is the Diagnosis?
Marc Imhotep Cray, MD
Pelvic Inflammatory Disease
158
Etiology and Epidemiology: Common causes include Chlamydia trachomatis
(subacute), Neisseria gonorrhoeae (acute), Gardnerella vaginalis, and
Trichomonas vaginalis; Usually occurs in young, nulliparous, sexually active
women with multiple partners
Pathology: Fallopian tubes: Edematous tubal serosa with fibrin covering;
purulent exudate in lumen; collections of pus may form a pyosalpinx or
degrading pus may form a hydrosalpinx (water-filled fallopian tubes)
Infection can also involve the ovaries and other pelvic structures
Marc Imhotep Cray, MD
PID cont.
159
Clinical Manifestations: High fever; lower abdominal pain; cervical motion
tenderness (chandelier sign); purulent cervical discharge; RUQ pain indicates
perihepatitis (Fitz-Hugh-Curtis syndrome); Salpingitis is a risk factor for ectopic
pregnancy, infertility, chronic pelvic pain, and adhesions
Treatment: Antibiotics effective against causative organism
Note: Ectopic pregnancy occurs most often in fallopian tubes, but may also
occur in ovary, abdominal cavity, or cervix
 Risk factors include salpingitis, endometriosis, and tubal ligation
 Clinical manifestations include severe abdominal pain 6 weeks after LMP
and elevated hCG which is lower than normal for pregnancy stage
Marc Imhotep Cray, MD
Pathogenesis of gonococcal infections
160
 Neisseria gonorrhoeae is a gram-negative diplococcus whose
surface pili form a barrier against phagocytosis by neutrophils
 Pili contain an immunoglobulin A (IgA) protease that digests
IgA on luminal surface of mucous membranes of urethra,
endocervix and fallopian tube thereby facilitating
attachment of gonococci
 Gonococci cause endocervicitis, vaginitis and salpingitis
 In men, gonococci attached to mucous membrane of urethra
cause urethritis and, sometimes, urethral stricture
 GC may also attach to sperm heads and be carried into the
fallopian tube Penetration of mucous membrane by
gonococci leads to stricture of fallopian tube, pelvic
inflammatory disease (PID) or tuboovarian abscess
Rubin R and Strayer DS Eds. Rubin’s Pathology: Clinicopathologic Foundations of
Medicine, 6th Ed. Baltimore: Lippincott Williams & Wilkins, 2012; Fig. 9.18, 394.
Click for large view
Marc Imhotep Cray, MD 161
Gonorrhea of the fallopian tube
Cross-section of a “pus tube” shows
thickening of wall and a lumen swollen
with pus
Rubin R and Strayer DS Eds. Rubin’s Pathology: Clinicopathologic Foundations of
Medicine, 6th Ed. Baltimore: Lippincott Williams & Wilkins, 2012; Fig. 9.19, 394.
Marc Imhotep Cray, MD 162
Acute salpingitis, microscopic
Klatt EC. Robbins and Cotran Atlas of Pathology, 3rd Ed. 2015.
Marc Imhotep Cray, MD 163
Tubo-ovarian abscess, gross
Klatt EC. Robbins and Cotran Atlas of Pathology, 3rd Ed. 2015.
Marc Imhotep Cray, MD 164
Ectopic pregnancy, gross
Klatt EC. Robbins and Cotran Atlas of Pathology, 3rd Ed. 2015.
Marc Imhotep Cray, MD
Vignette #16
165
After vaginally delivering a newborn from a 32-year-old woman, you
find the mother to be hemorrhaging an abnormally large amount of
blood. You recall that she has had one previous C-section and
extensive scarring. Given the amount of blood after delivery and the
abnormal gross appearance of the placental remnants, you believe
that the patient’s pregnancy was complicated by a defective decidual
layer that allowed the placenta to attach directly to the myometrium.
You call the operating room to set up for an emergency hysterectomy,
which you believe may be necessary to control the patient’s massive
hemorrhage.
What is the Diagnosis?
Marc Imhotep Cray, MD
Placental Attachment Abnormalities (Abruptio Placentae,
Placenta Previa, Placenta Accreta)
166
Etiology:
Abruptio placentae: May be associated with DIC; increased risk with smoking,
cocaine, and hypertension
Placenta accreta: Predisposed by prior C-section scars or endometrial
inflammation
Placenta previa: Predisposed by prior C-section scars
Pathophysiology:
Abruptio placentae: Premature separation of placenta
Placenta accreta: Defective decidual layer allows placenta to attach directly to
myometrium
Placenta previa: Attachment of placenta to lower uterine segment or cervix;
may occlude cervical os; may coexist with placenta accreta
Marc Imhotep Cray, MD
Placental Attachment Abnormalities cont.
167
Clinical Manifestations:
Abruptio placentae: Painful uterine bleeding usually during third trimester; can
result in fetal death
Placenta accreta: Massive hemorrhage after delivery
Placenta previa: Painless bleeding in any trimester; premature labor
Treatment:
Abruptio placentae: Immediate fetal delivery; control of maternal bleeding
Placenta accreta: Hysterectomy may be necessary to stop bleeding
Placenta previa: Bed rest; possible hospitalization
Marc Imhotep Cray, MD
Vignette #17
168
A 31-year-old pregnant woman is brought to the emergency room
complaining of headaches and blurred vision of 1-week duration. She
is currently 32 weeks into her first pregnancy. On physical
examination, you find edema of the face and lower extremities. Her
blood pressure is 160/100, but she has no prior history of
hypertension. Laboratory studies show a mild thrombocytopenia,
elevated AST and ALT, and significant proteinuria. Based on these
findings, you admit the patient for immediate bed rest, close
monitoring, and blood pressure control. You tell the patient that it is
possible that you may need to deliver her baby early in order to treat
the patient’s current condition.
What is the Diagnosis?
Marc Imhotep Cray, MD
Preeclampsia and Eclampsia
169
Etiology and Epidemiology: Risk factors include preexisting hypertension,
diabetes, chronic renal disease, autoimmune disorders, or twin gestation
Affects 7% of pregnant women usually during the third trimester of a woman’s
first pregnancy
Pathology and Pathophysiology: Placenta: Evidence of infarct; presence of
retroplacental hematomas; decreased vascularity; fibrinoid necrosis
 Pathophysiology: Intrinsic defect in invading cytotrophoblast leads to
remodeling of uterine vasculature and resulting placental ischemia;
decreased placental perfusion induces vasoconstrictive effects leading to
toxemic hypertension in susceptible women
Marc Imhotep Cray, MD
Preeclampsia and Eclampsia cont.
170
Clinical Manifestations: Preeclampsia: Triad of hypertension, proteinuria, and
edema; associated Sx include headache, blurry vision, and abdominal pain
Eclampsia: Preeclampsia triad plus seizures, altered mentation, hyperreflexia,
and possibly DIC
Lab findings: Thrombocytopenia, elevated LFTs, hyperuricemia, hemolytic anemia
Treatment: Preeclampsia: Delivery of fetus as soon as possible; bed rest; salt
restriction; treatment of hypertension
Eclampsia: Medical emergency; treat with IV magnesium sulfate and diazepam;
delivery of fetus
NB: HELLP syndrome refers to hemolysis, elevated LFTs, and low platelets and is a
severe form of preeclampsia (Video on HELLP syndrome & “USMLE”)
Marc Imhotep Cray, MD
THE END
See next slide for further study.
171
Marc Imhotep Cray, MD
Companion Tools and Resources (online)
172
Discipline (Pathology) Track Folder
 Reproductive Pathology
Organ Systems Track Folders
 M and FM Reproductive System & FM Breast
 Pregnancy, Childbirth, & the Puerperium
Textbooks:
Kumar V and Abbas AK. Robbins and Cotran Basis Pathology, 10th Ed.
Philadelphia: Saunders, 2018.
Rubin R and Strayer DS Eds. Rubin’s Essential of Pathology:, 6th Ed.
Baltimore: Lippincott Williams & Wilkins, 2014.

More Related Content

What's hot

Disorders of pregnancy and placental pathology
Disorders of pregnancy and placental pathologyDisorders of pregnancy and placental pathology
Disorders of pregnancy and placental pathologyDr Prakriti Shukla
 
Breast pathology 1
Breast pathology 1Breast pathology 1
Breast pathology 1Prasad CSBR
 
Basics of Pancreatitis and Pancreatic Exocrine Insufficiency
Basics of Pancreatitis and Pancreatic Exocrine InsufficiencyBasics of Pancreatitis and Pancreatic Exocrine Insufficiency
Basics of Pancreatitis and Pancreatic Exocrine InsufficiencyAjin Pisharody
 
Diasease of small intestine
Diasease of small intestineDiasease of small intestine
Diasease of small intestineNur Idris
 
DISORDERS OF LIVER, PANCREAS AND BILIARY TRACT
DISORDERS OF LIVER, PANCREAS AND BILIARY TRACTDISORDERS OF LIVER, PANCREAS AND BILIARY TRACT
DISORDERS OF LIVER, PANCREAS AND BILIARY TRACTAndhrapradesh
 
Congenital anamalies of biliary system aryaja
Congenital anamalies of biliary system aryajaCongenital anamalies of biliary system aryaja
Congenital anamalies of biliary system aryajaRamesh Bhat
 
Git pathology lecture
Git pathology lectureGit pathology lecture
Git pathology lectureDr Ashish Jha
 
Benign and Malignant Breast Diseases
Benign and Malignant Breast DiseasesBenign and Malignant Breast Diseases
Benign and Malignant Breast Diseasesyuyuricci
 
Meckelsdiverticulum
MeckelsdiverticulumMeckelsdiverticulum
Meckelsdiverticulumcoolboy101pk
 
Chronic cholecystitis & Jaundice
Chronic cholecystitis & JaundiceChronic cholecystitis & Jaundice
Chronic cholecystitis & JaundiceMuhammad Eimaduddin
 
Stomach pathology lecture
Stomach pathology lectureStomach pathology lecture
Stomach pathology lectureDrsapna Harsha
 
Gastrointestinal cancer
Gastrointestinal cancerGastrointestinal cancer
Gastrointestinal canceramakolslide
 

What's hot (20)

GESTOSES part1
GESTOSES part1GESTOSES part1
GESTOSES part1
 
Disorders of pregnancy and placental pathology
Disorders of pregnancy and placental pathologyDisorders of pregnancy and placental pathology
Disorders of pregnancy and placental pathology
 
Breast pathology 1
Breast pathology 1Breast pathology 1
Breast pathology 1
 
Git pathology m scyear2011 12
Git pathology m scyear2011 12Git pathology m scyear2011 12
Git pathology m scyear2011 12
 
Diseases of git
Diseases of gitDiseases of git
Diseases of git
 
Pathology of Breast Disorders
Pathology of Breast DisordersPathology of Breast Disorders
Pathology of Breast Disorders
 
Basics of Pancreatitis and Pancreatic Exocrine Insufficiency
Basics of Pancreatitis and Pancreatic Exocrine InsufficiencyBasics of Pancreatitis and Pancreatic Exocrine Insufficiency
Basics of Pancreatitis and Pancreatic Exocrine Insufficiency
 
Pathology of the Esophagus
Pathology of the EsophagusPathology of the Esophagus
Pathology of the Esophagus
 
Diasease of small intestine
Diasease of small intestineDiasease of small intestine
Diasease of small intestine
 
DISORDERS OF LIVER, PANCREAS AND BILIARY TRACT
DISORDERS OF LIVER, PANCREAS AND BILIARY TRACTDISORDERS OF LIVER, PANCREAS AND BILIARY TRACT
DISORDERS OF LIVER, PANCREAS AND BILIARY TRACT
 
Pathophysiology of liver
Pathophysiology of liverPathophysiology of liver
Pathophysiology of liver
 
Congenital anamalies of biliary system aryaja
Congenital anamalies of biliary system aryajaCongenital anamalies of biliary system aryaja
Congenital anamalies of biliary system aryaja
 
Pathology Of Kidney
Pathology Of KidneyPathology Of Kidney
Pathology Of Kidney
 
Git pathology lecture
Git pathology lectureGit pathology lecture
Git pathology lecture
 
Benign and Malignant Breast Diseases
Benign and Malignant Breast DiseasesBenign and Malignant Breast Diseases
Benign and Malignant Breast Diseases
 
Meckelsdiverticulum
MeckelsdiverticulumMeckelsdiverticulum
Meckelsdiverticulum
 
Chronic cholecystitis & Jaundice
Chronic cholecystitis & JaundiceChronic cholecystitis & Jaundice
Chronic cholecystitis & Jaundice
 
Stomach pathology lecture
Stomach pathology lectureStomach pathology lecture
Stomach pathology lecture
 
Gastrointestinal cancer
Gastrointestinal cancerGastrointestinal cancer
Gastrointestinal cancer
 
Inflammatory bowel disease
Inflammatory bowel diseaseInflammatory bowel disease
Inflammatory bowel disease
 

Similar to Reproductive System Pathology_FM Breast and FM Reproductive Systems

Oncologic Pathology_A Case-based Organ Systems Review (USMLE Step 1)
Oncologic Pathology_A Case-based Organ Systems Review (USMLE Step 1)Oncologic Pathology_A Case-based Organ Systems Review (USMLE Step 1)
Oncologic Pathology_A Case-based Organ Systems Review (USMLE Step 1)Imhotep Virtual Medical School
 
Breast Cancer Congress 2018 | New York | USA | 25-26 May | about Breast cance...
Breast Cancer Congress 2018 | New York | USA | 25-26 May | about Breast cance...Breast Cancer Congress 2018 | New York | USA | 25-26 May | about Breast cance...
Breast Cancer Congress 2018 | New York | USA | 25-26 May | about Breast cance...Paul Hederson
 
Determinants of Malignant Transformation in Fibrocystic Disease of Breast
Determinants of Malignant Transformation in Fibrocystic Disease of BreastDeterminants of Malignant Transformation in Fibrocystic Disease of Breast
Determinants of Malignant Transformation in Fibrocystic Disease of BreastKETAN VAGHOLKAR
 
Tubular Adenoma of the Breast: A Rare Case Presentation and Review of Literat...
Tubular Adenoma of the Breast: A Rare Case Presentation and Review of Literat...Tubular Adenoma of the Breast: A Rare Case Presentation and Review of Literat...
Tubular Adenoma of the Breast: A Rare Case Presentation and Review of Literat...Crimsonpublisherssmoaj
 
Molecular Portrait of the Normal Human Breast Tissue and Its Influence on Bre...
Molecular Portrait of the Normal Human Breast Tissue and Its Influence on Bre...Molecular Portrait of the Normal Human Breast Tissue and Its Influence on Bre...
Molecular Portrait of the Normal Human Breast Tissue and Its Influence on Bre...Mădălin Margan
 
Reproductive System Pathology_Male Reproductive Systems
Reproductive System Pathology_Male Reproductive SystemsReproductive System Pathology_Male Reproductive Systems
Reproductive System Pathology_Male Reproductive SystemsImhotep Virtual Medical School
 
23204937
2320493723204937
23204937radgirl
 
Pathophysiology of breast cancer
Pathophysiology of breast cancerPathophysiology of breast cancer
Pathophysiology of breast cancerPriyanka Padhy
 
Imaging of breast hamartomas
Imaging of breast hamartomasImaging of breast hamartomas
Imaging of breast hamartomasAI Publications
 
Horological Diagnosis of Cervical cancer.pptx
Horological Diagnosis of Cervical cancer.pptxHorological Diagnosis of Cervical cancer.pptx
Horological Diagnosis of Cervical cancer.pptxAbdulmuminIliyasu
 
Breast cancer
Breast cancerBreast cancer
Breast cancersanal
 
Genesilencing in Breast Cancer
Genesilencing in Breast CancerGenesilencing in Breast Cancer
Genesilencing in Breast CancerTamil Jothi
 
Gene silencing in Breast cancer
Gene silencing in Breast cancer Gene silencing in Breast cancer
Gene silencing in Breast cancer Santhi Dasari
 
Clinical presentation and investigations for breast carcinoma
Clinical presentation and investigations for breast carcinomaClinical presentation and investigations for breast carcinoma
Clinical presentation and investigations for breast carcinomaViswa Kumar
 

Similar to Reproductive System Pathology_FM Breast and FM Reproductive Systems (20)

Oncologic Pathology_A Case-based Organ Systems Review (USMLE Step 1)
Oncologic Pathology_A Case-based Organ Systems Review (USMLE Step 1)Oncologic Pathology_A Case-based Organ Systems Review (USMLE Step 1)
Oncologic Pathology_A Case-based Organ Systems Review (USMLE Step 1)
 
Molecular Subtypes of Breast Cancer
Molecular Subtypes of Breast CancerMolecular Subtypes of Breast Cancer
Molecular Subtypes of Breast Cancer
 
Chapter 15
Chapter 15Chapter 15
Chapter 15
 
Breast Cancer Congress 2018 | New York | USA | 25-26 May | about Breast cance...
Breast Cancer Congress 2018 | New York | USA | 25-26 May | about Breast cance...Breast Cancer Congress 2018 | New York | USA | 25-26 May | about Breast cance...
Breast Cancer Congress 2018 | New York | USA | 25-26 May | about Breast cance...
 
12 breast cancer
12 breast cancer12 breast cancer
12 breast cancer
 
Determinants of Malignant Transformation in Fibrocystic Disease of Breast
Determinants of Malignant Transformation in Fibrocystic Disease of BreastDeterminants of Malignant Transformation in Fibrocystic Disease of Breast
Determinants of Malignant Transformation in Fibrocystic Disease of Breast
 
Tubular Adenoma of the Breast: A Rare Case Presentation and Review of Literat...
Tubular Adenoma of the Breast: A Rare Case Presentation and Review of Literat...Tubular Adenoma of the Breast: A Rare Case Presentation and Review of Literat...
Tubular Adenoma of the Breast: A Rare Case Presentation and Review of Literat...
 
Molecular Portrait of the Normal Human Breast Tissue and Its Influence on Bre...
Molecular Portrait of the Normal Human Breast Tissue and Its Influence on Bre...Molecular Portrait of the Normal Human Breast Tissue and Its Influence on Bre...
Molecular Portrait of the Normal Human Breast Tissue and Its Influence on Bre...
 
Types & Sub-types of Breast Cancer
Types & Sub-types of Breast CancerTypes & Sub-types of Breast Cancer
Types & Sub-types of Breast Cancer
 
Breast cancer
Breast cancerBreast cancer
Breast cancer
 
Reproductive System Pathology_Male Reproductive Systems
Reproductive System Pathology_Male Reproductive SystemsReproductive System Pathology_Male Reproductive Systems
Reproductive System Pathology_Male Reproductive Systems
 
23204937
2320493723204937
23204937
 
Pathophysiology of breast cancer
Pathophysiology of breast cancerPathophysiology of breast cancer
Pathophysiology of breast cancer
 
Breast Cancer Causes & Dagnosis
Breast Cancer Causes & DagnosisBreast Cancer Causes & Dagnosis
Breast Cancer Causes & Dagnosis
 
Imaging of breast hamartomas
Imaging of breast hamartomasImaging of breast hamartomas
Imaging of breast hamartomas
 
Horological Diagnosis of Cervical cancer.pptx
Horological Diagnosis of Cervical cancer.pptxHorological Diagnosis of Cervical cancer.pptx
Horological Diagnosis of Cervical cancer.pptx
 
Breast cancer
Breast cancerBreast cancer
Breast cancer
 
Genesilencing in Breast Cancer
Genesilencing in Breast CancerGenesilencing in Breast Cancer
Genesilencing in Breast Cancer
 
Gene silencing in Breast cancer
Gene silencing in Breast cancer Gene silencing in Breast cancer
Gene silencing in Breast cancer
 
Clinical presentation and investigations for breast carcinoma
Clinical presentation and investigations for breast carcinomaClinical presentation and investigations for breast carcinoma
Clinical presentation and investigations for breast carcinoma
 

More from Imhotep Virtual Medical School

Nervous System Pathology_A Case-based Learning Approach
Nervous System Pathology_A Case-based Learning ApproachNervous System Pathology_A Case-based Learning Approach
Nervous System Pathology_A Case-based Learning ApproachImhotep Virtual Medical School
 
CVS Function, Regulation of the Heart and Overview of Therapeutic Goals in CV...
CVS Function, Regulation of the Heart and Overview of Therapeutic Goals in CV...CVS Function, Regulation of the Heart and Overview of Therapeutic Goals in CV...
CVS Function, Regulation of the Heart and Overview of Therapeutic Goals in CV...Imhotep Virtual Medical School
 
Cardiovascular Pathology Case-based_Gross and Microscopic
Cardiovascular Pathology Case-based_Gross and MicroscopicCardiovascular Pathology Case-based_Gross and Microscopic
Cardiovascular Pathology Case-based_Gross and MicroscopicImhotep Virtual Medical School
 
Clinical Pharmacology for Medical Students_USMLE Step 1 & 2 Review
Clinical Pharmacology for Medical Students_USMLE Step 1 & 2 ReviewClinical Pharmacology for Medical Students_USMLE Step 1 & 2 Review
Clinical Pharmacology for Medical Students_USMLE Step 1 & 2 ReviewImhotep Virtual Medical School
 
Make the Dx_ A Case-based Intro to Select Cardiovascular and Respiratory Dise...
Make the Dx_ A Case-based Intro to Select Cardiovascular and Respiratory Dise...Make the Dx_ A Case-based Intro to Select Cardiovascular and Respiratory Dise...
Make the Dx_ A Case-based Intro to Select Cardiovascular and Respiratory Dise...Imhotep Virtual Medical School
 
Myocardial infarction_ Causes, Symptoms, Diagnosis, Treatment, and Pathology
Myocardial infarction_ Causes, Symptoms, Diagnosis, Treatment, and PathologyMyocardial infarction_ Causes, Symptoms, Diagnosis, Treatment, and Pathology
Myocardial infarction_ Causes, Symptoms, Diagnosis, Treatment, and PathologyImhotep Virtual Medical School
 
Autonomic Nervous System Physiology and Pharmacology_Overview| Review of ANS
Autonomic Nervous System Physiology and Pharmacology_Overview| Review of ANSAutonomic Nervous System Physiology and Pharmacology_Overview| Review of ANS
Autonomic Nervous System Physiology and Pharmacology_Overview| Review of ANSImhotep Virtual Medical School
 
Evidence-Based Physical Diagnosis_Lect. 1_ What is Evidence-Based Physical Di...
Evidence-Based Physical Diagnosis_Lect. 1_ What is Evidence-Based Physical Di...Evidence-Based Physical Diagnosis_Lect. 1_ What is Evidence-Based Physical Di...
Evidence-Based Physical Diagnosis_Lect. 1_ What is Evidence-Based Physical Di...Imhotep Virtual Medical School
 

More from Imhotep Virtual Medical School (20)

Pathology and Pathophysiology of Shock
Pathology and Pathophysiology of ShockPathology and Pathophysiology of Shock
Pathology and Pathophysiology of Shock
 
Drugs Used In Disorders of the Reproductive System
Drugs Used In Disorders of the Reproductive SystemDrugs Used In Disorders of the Reproductive System
Drugs Used In Disorders of the Reproductive System
 
Nervous System Pathology_A Case-based Learning Approach
Nervous System Pathology_A Case-based Learning ApproachNervous System Pathology_A Case-based Learning Approach
Nervous System Pathology_A Case-based Learning Approach
 
CVS Function, Regulation of the Heart and Overview of Therapeutic Goals in CV...
CVS Function, Regulation of the Heart and Overview of Therapeutic Goals in CV...CVS Function, Regulation of the Heart and Overview of Therapeutic Goals in CV...
CVS Function, Regulation of the Heart and Overview of Therapeutic Goals in CV...
 
Cardiovascular Pathology Case-based_Gross and Microscopic
Cardiovascular Pathology Case-based_Gross and MicroscopicCardiovascular Pathology Case-based_Gross and Microscopic
Cardiovascular Pathology Case-based_Gross and Microscopic
 
HIV / AIDS Pathology
HIV / AIDS PathologyHIV / AIDS Pathology
HIV / AIDS Pathology
 
Sepsis & Septic Shock
Sepsis & Septic ShockSepsis & Septic Shock
Sepsis & Septic Shock
 
Drugs Used in infectious Disease_Antibiotics
Drugs Used in infectious Disease_AntibioticsDrugs Used in infectious Disease_Antibiotics
Drugs Used in infectious Disease_Antibiotics
 
Hematopoietic and Lymphoid Systems Pathology
Hematopoietic and Lymphoid Systems  PathologyHematopoietic and Lymphoid Systems  Pathology
Hematopoietic and Lymphoid Systems Pathology
 
Drugs Used in Neoplastic Disorders
Drugs Used in Neoplastic DisordersDrugs Used in Neoplastic Disorders
Drugs Used in Neoplastic Disorders
 
Neoplasia & Oncologic Pathology
Neoplasia & Oncologic PathologyNeoplasia & Oncologic Pathology
Neoplasia & Oncologic Pathology
 
Clinical Pharmacology for Medical Students_USMLE Step 1 & 2 Review
Clinical Pharmacology for Medical Students_USMLE Step 1 & 2 ReviewClinical Pharmacology for Medical Students_USMLE Step 1 & 2 Review
Clinical Pharmacology for Medical Students_USMLE Step 1 & 2 Review
 
Make the Dx_ A Case-based Intro to Select Cardiovascular and Respiratory Dise...
Make the Dx_ A Case-based Intro to Select Cardiovascular and Respiratory Dise...Make the Dx_ A Case-based Intro to Select Cardiovascular and Respiratory Dise...
Make the Dx_ A Case-based Intro to Select Cardiovascular and Respiratory Dise...
 
Myocardial infarction_ Causes, Symptoms, Diagnosis, Treatment, and Pathology
Myocardial infarction_ Causes, Symptoms, Diagnosis, Treatment, and PathologyMyocardial infarction_ Causes, Symptoms, Diagnosis, Treatment, and Pathology
Myocardial infarction_ Causes, Symptoms, Diagnosis, Treatment, and Pathology
 
Basic CXR Interpretation_Diagnostic Radiographs
Basic CXR Interpretation_Diagnostic RadiographsBasic CXR Interpretation_Diagnostic Radiographs
Basic CXR Interpretation_Diagnostic Radiographs
 
Electrocardiogram (ECG) Interpretation_Module 1 of 2
Electrocardiogram (ECG) Interpretation_Module 1 of 2Electrocardiogram (ECG) Interpretation_Module 1 of 2
Electrocardiogram (ECG) Interpretation_Module 1 of 2
 
Autonomic Nervous System Physiology and Pharmacology_Overview| Review of ANS
Autonomic Nervous System Physiology and Pharmacology_Overview| Review of ANSAutonomic Nervous System Physiology and Pharmacology_Overview| Review of ANS
Autonomic Nervous System Physiology and Pharmacology_Overview| Review of ANS
 
Evidence-Based Physical Diagnosis_Lect. 1_ What is Evidence-Based Physical Di...
Evidence-Based Physical Diagnosis_Lect. 1_ What is Evidence-Based Physical Di...Evidence-Based Physical Diagnosis_Lect. 1_ What is Evidence-Based Physical Di...
Evidence-Based Physical Diagnosis_Lect. 1_ What is Evidence-Based Physical Di...
 
Prescription Writing 101 for Medical Students
Prescription Writing 101 for Medical StudentsPrescription Writing 101 for Medical Students
Prescription Writing 101 for Medical Students
 
Admitting a Patient to the Hospital_Admit Orders
Admitting a Patient to the Hospital_Admit OrdersAdmitting a Patient to the Hospital_Admit Orders
Admitting a Patient to the Hospital_Admit Orders
 

Recently uploaded

Call Girls Jayanagar Just Call 7001305949 Top Class Call Girl Service Available
Call Girls Jayanagar Just Call 7001305949 Top Class Call Girl Service AvailableCall Girls Jayanagar Just Call 7001305949 Top Class Call Girl Service Available
Call Girls Jayanagar Just Call 7001305949 Top Class Call Girl Service Availablenarwatsonia7
 
Low Rate Call Girls Pune Esha 9907093804 Short 1500 Night 6000 Best call girl...
Low Rate Call Girls Pune Esha 9907093804 Short 1500 Night 6000 Best call girl...Low Rate Call Girls Pune Esha 9907093804 Short 1500 Night 6000 Best call girl...
Low Rate Call Girls Pune Esha 9907093804 Short 1500 Night 6000 Best call girl...Miss joya
 
Call Girls Electronic City Just Call 7001305949 Top Class Call Girl Service A...
Call Girls Electronic City Just Call 7001305949 Top Class Call Girl Service A...Call Girls Electronic City Just Call 7001305949 Top Class Call Girl Service A...
Call Girls Electronic City Just Call 7001305949 Top Class Call Girl Service A...narwatsonia7
 
VIP Call Girls Pune Vrinda 9907093804 Short 1500 Night 6000 Best call girls S...
VIP Call Girls Pune Vrinda 9907093804 Short 1500 Night 6000 Best call girls S...VIP Call Girls Pune Vrinda 9907093804 Short 1500 Night 6000 Best call girls S...
VIP Call Girls Pune Vrinda 9907093804 Short 1500 Night 6000 Best call girls S...Miss joya
 
Housewife Call Girls Bangalore - Call 7001305949 Rs-3500 with A/C Room Cash o...
Housewife Call Girls Bangalore - Call 7001305949 Rs-3500 with A/C Room Cash o...Housewife Call Girls Bangalore - Call 7001305949 Rs-3500 with A/C Room Cash o...
Housewife Call Girls Bangalore - Call 7001305949 Rs-3500 with A/C Room Cash o...narwatsonia7
 
Aspirin presentation slides by Dr. Rewas Ali
Aspirin presentation slides by Dr. Rewas AliAspirin presentation slides by Dr. Rewas Ali
Aspirin presentation slides by Dr. Rewas AliRewAs ALI
 
Call Girls Service in Bommanahalli - 7001305949 with real photos and phone nu...
Call Girls Service in Bommanahalli - 7001305949 with real photos and phone nu...Call Girls Service in Bommanahalli - 7001305949 with real photos and phone nu...
Call Girls Service in Bommanahalli - 7001305949 with real photos and phone nu...narwatsonia7
 
VIP Call Girls Lucknow Nandini 7001305949 Independent Escort Service Lucknow
VIP Call Girls Lucknow Nandini 7001305949 Independent Escort Service LucknowVIP Call Girls Lucknow Nandini 7001305949 Independent Escort Service Lucknow
VIP Call Girls Lucknow Nandini 7001305949 Independent Escort Service Lucknownarwatsonia7
 
Book Call Girls in Kasavanahalli - 7001305949 with real photos and phone numbers
Book Call Girls in Kasavanahalli - 7001305949 with real photos and phone numbersBook Call Girls in Kasavanahalli - 7001305949 with real photos and phone numbers
Book Call Girls in Kasavanahalli - 7001305949 with real photos and phone numbersnarwatsonia7
 
Bangalore Call Girls Majestic 📞 9907093804 High Profile Service 100% Safe
Bangalore Call Girls Majestic 📞 9907093804 High Profile Service 100% SafeBangalore Call Girls Majestic 📞 9907093804 High Profile Service 100% Safe
Bangalore Call Girls Majestic 📞 9907093804 High Profile Service 100% Safenarwatsonia7
 
College Call Girls Pune Mira 9907093804 Short 1500 Night 6000 Best call girls...
College Call Girls Pune Mira 9907093804 Short 1500 Night 6000 Best call girls...College Call Girls Pune Mira 9907093804 Short 1500 Night 6000 Best call girls...
College Call Girls Pune Mira 9907093804 Short 1500 Night 6000 Best call girls...Miss joya
 
Russian Call Girls in Pune Riya 9907093804 Short 1500 Night 6000 Best call gi...
Russian Call Girls in Pune Riya 9907093804 Short 1500 Night 6000 Best call gi...Russian Call Girls in Pune Riya 9907093804 Short 1500 Night 6000 Best call gi...
Russian Call Girls in Pune Riya 9907093804 Short 1500 Night 6000 Best call gi...Miss joya
 
Call Girl Surat Madhuri 7001305949 Independent Escort Service Surat
Call Girl Surat Madhuri 7001305949 Independent Escort Service SuratCall Girl Surat Madhuri 7001305949 Independent Escort Service Surat
Call Girl Surat Madhuri 7001305949 Independent Escort Service Suratnarwatsonia7
 
Low Rate Call Girls Mumbai Suman 9910780858 Independent Escort Service Mumbai
Low Rate Call Girls Mumbai Suman 9910780858 Independent Escort Service MumbaiLow Rate Call Girls Mumbai Suman 9910780858 Independent Escort Service Mumbai
Low Rate Call Girls Mumbai Suman 9910780858 Independent Escort Service Mumbaisonalikaur4
 
Call Girls Service In Shyam Nagar Whatsapp 8445551418 Independent Escort Service
Call Girls Service In Shyam Nagar Whatsapp 8445551418 Independent Escort ServiceCall Girls Service In Shyam Nagar Whatsapp 8445551418 Independent Escort Service
Call Girls Service In Shyam Nagar Whatsapp 8445551418 Independent Escort Serviceparulsinha
 
Call Girls Service Nandiambakkam | 7001305949 At Low Cost Cash Payment Booking
Call Girls Service Nandiambakkam | 7001305949 At Low Cost Cash Payment BookingCall Girls Service Nandiambakkam | 7001305949 At Low Cost Cash Payment Booking
Call Girls Service Nandiambakkam | 7001305949 At Low Cost Cash Payment BookingNehru place Escorts
 
Call Girls Hebbal Just Call 7001305949 Top Class Call Girl Service Available
Call Girls Hebbal Just Call 7001305949 Top Class Call Girl Service AvailableCall Girls Hebbal Just Call 7001305949 Top Class Call Girl Service Available
Call Girls Hebbal Just Call 7001305949 Top Class Call Girl Service Availablenarwatsonia7
 
Call Girls Budhwar Peth 7001305949 All Area Service COD available Any Time
Call Girls Budhwar Peth 7001305949 All Area Service COD available Any TimeCall Girls Budhwar Peth 7001305949 All Area Service COD available Any Time
Call Girls Budhwar Peth 7001305949 All Area Service COD available Any Timevijaych2041
 
Asthma Review - GINA guidelines summary 2024
Asthma Review - GINA guidelines summary 2024Asthma Review - GINA guidelines summary 2024
Asthma Review - GINA guidelines summary 2024Gabriel Guevara MD
 

Recently uploaded (20)

Call Girls Jayanagar Just Call 7001305949 Top Class Call Girl Service Available
Call Girls Jayanagar Just Call 7001305949 Top Class Call Girl Service AvailableCall Girls Jayanagar Just Call 7001305949 Top Class Call Girl Service Available
Call Girls Jayanagar Just Call 7001305949 Top Class Call Girl Service Available
 
Low Rate Call Girls Pune Esha 9907093804 Short 1500 Night 6000 Best call girl...
Low Rate Call Girls Pune Esha 9907093804 Short 1500 Night 6000 Best call girl...Low Rate Call Girls Pune Esha 9907093804 Short 1500 Night 6000 Best call girl...
Low Rate Call Girls Pune Esha 9907093804 Short 1500 Night 6000 Best call girl...
 
Call Girls Electronic City Just Call 7001305949 Top Class Call Girl Service A...
Call Girls Electronic City Just Call 7001305949 Top Class Call Girl Service A...Call Girls Electronic City Just Call 7001305949 Top Class Call Girl Service A...
Call Girls Electronic City Just Call 7001305949 Top Class Call Girl Service A...
 
VIP Call Girls Pune Vrinda 9907093804 Short 1500 Night 6000 Best call girls S...
VIP Call Girls Pune Vrinda 9907093804 Short 1500 Night 6000 Best call girls S...VIP Call Girls Pune Vrinda 9907093804 Short 1500 Night 6000 Best call girls S...
VIP Call Girls Pune Vrinda 9907093804 Short 1500 Night 6000 Best call girls S...
 
Housewife Call Girls Bangalore - Call 7001305949 Rs-3500 with A/C Room Cash o...
Housewife Call Girls Bangalore - Call 7001305949 Rs-3500 with A/C Room Cash o...Housewife Call Girls Bangalore - Call 7001305949 Rs-3500 with A/C Room Cash o...
Housewife Call Girls Bangalore - Call 7001305949 Rs-3500 with A/C Room Cash o...
 
Aspirin presentation slides by Dr. Rewas Ali
Aspirin presentation slides by Dr. Rewas AliAspirin presentation slides by Dr. Rewas Ali
Aspirin presentation slides by Dr. Rewas Ali
 
Call Girls Service in Bommanahalli - 7001305949 with real photos and phone nu...
Call Girls Service in Bommanahalli - 7001305949 with real photos and phone nu...Call Girls Service in Bommanahalli - 7001305949 with real photos and phone nu...
Call Girls Service in Bommanahalli - 7001305949 with real photos and phone nu...
 
VIP Call Girls Lucknow Nandini 7001305949 Independent Escort Service Lucknow
VIP Call Girls Lucknow Nandini 7001305949 Independent Escort Service LucknowVIP Call Girls Lucknow Nandini 7001305949 Independent Escort Service Lucknow
VIP Call Girls Lucknow Nandini 7001305949 Independent Escort Service Lucknow
 
Book Call Girls in Kasavanahalli - 7001305949 with real photos and phone numbers
Book Call Girls in Kasavanahalli - 7001305949 with real photos and phone numbersBook Call Girls in Kasavanahalli - 7001305949 with real photos and phone numbers
Book Call Girls in Kasavanahalli - 7001305949 with real photos and phone numbers
 
Bangalore Call Girls Majestic 📞 9907093804 High Profile Service 100% Safe
Bangalore Call Girls Majestic 📞 9907093804 High Profile Service 100% SafeBangalore Call Girls Majestic 📞 9907093804 High Profile Service 100% Safe
Bangalore Call Girls Majestic 📞 9907093804 High Profile Service 100% Safe
 
College Call Girls Pune Mira 9907093804 Short 1500 Night 6000 Best call girls...
College Call Girls Pune Mira 9907093804 Short 1500 Night 6000 Best call girls...College Call Girls Pune Mira 9907093804 Short 1500 Night 6000 Best call girls...
College Call Girls Pune Mira 9907093804 Short 1500 Night 6000 Best call girls...
 
Russian Call Girls in Pune Riya 9907093804 Short 1500 Night 6000 Best call gi...
Russian Call Girls in Pune Riya 9907093804 Short 1500 Night 6000 Best call gi...Russian Call Girls in Pune Riya 9907093804 Short 1500 Night 6000 Best call gi...
Russian Call Girls in Pune Riya 9907093804 Short 1500 Night 6000 Best call gi...
 
Call Girl Surat Madhuri 7001305949 Independent Escort Service Surat
Call Girl Surat Madhuri 7001305949 Independent Escort Service SuratCall Girl Surat Madhuri 7001305949 Independent Escort Service Surat
Call Girl Surat Madhuri 7001305949 Independent Escort Service Surat
 
Low Rate Call Girls Mumbai Suman 9910780858 Independent Escort Service Mumbai
Low Rate Call Girls Mumbai Suman 9910780858 Independent Escort Service MumbaiLow Rate Call Girls Mumbai Suman 9910780858 Independent Escort Service Mumbai
Low Rate Call Girls Mumbai Suman 9910780858 Independent Escort Service Mumbai
 
Call Girls Service In Shyam Nagar Whatsapp 8445551418 Independent Escort Service
Call Girls Service In Shyam Nagar Whatsapp 8445551418 Independent Escort ServiceCall Girls Service In Shyam Nagar Whatsapp 8445551418 Independent Escort Service
Call Girls Service In Shyam Nagar Whatsapp 8445551418 Independent Escort Service
 
Call Girls Service Nandiambakkam | 7001305949 At Low Cost Cash Payment Booking
Call Girls Service Nandiambakkam | 7001305949 At Low Cost Cash Payment BookingCall Girls Service Nandiambakkam | 7001305949 At Low Cost Cash Payment Booking
Call Girls Service Nandiambakkam | 7001305949 At Low Cost Cash Payment Booking
 
Escort Service Call Girls In Sarita Vihar,, 99530°56974 Delhi NCR
Escort Service Call Girls In Sarita Vihar,, 99530°56974 Delhi NCREscort Service Call Girls In Sarita Vihar,, 99530°56974 Delhi NCR
Escort Service Call Girls In Sarita Vihar,, 99530°56974 Delhi NCR
 
Call Girls Hebbal Just Call 7001305949 Top Class Call Girl Service Available
Call Girls Hebbal Just Call 7001305949 Top Class Call Girl Service AvailableCall Girls Hebbal Just Call 7001305949 Top Class Call Girl Service Available
Call Girls Hebbal Just Call 7001305949 Top Class Call Girl Service Available
 
Call Girls Budhwar Peth 7001305949 All Area Service COD available Any Time
Call Girls Budhwar Peth 7001305949 All Area Service COD available Any TimeCall Girls Budhwar Peth 7001305949 All Area Service COD available Any Time
Call Girls Budhwar Peth 7001305949 All Area Service COD available Any Time
 
Asthma Review - GINA guidelines summary 2024
Asthma Review - GINA guidelines summary 2024Asthma Review - GINA guidelines summary 2024
Asthma Review - GINA guidelines summary 2024
 

Reproductive System Pathology_FM Breast and FM Reproductive Systems

  • 1. Marc Imhotep Cray, MD Photo: Color-enhanced scanning electron micrograph of a human spermatocyte fertilizing an oocyte; From: Seeley’s Anatomy & Physiology 10th ed New York, NY: McGraw-Hill 2010
  • 2. Marc Imhotep Cray, MD Learning Outcomes cont. 2 Breast: 1. List the inflammatory breast lesions and discuss their etiology. 2. Describe the morphology of the inflammatory breast lesions with special reference to breast abscess, duct ectasia and fat necrosis. 3. List the proliferative breast lesions and discuss their etiology. 4. Describe the morphology of the proliferative breast lesions with special reference to fibrocystic changes. 5. Explain the relationship between fibrocystic changes of the breast and breast cancer. 6. Classify breast tumors By the end of this sequence the learner will/should be able to:
  • 3. Marc Imhotep Cray, MD Female Breast Learning Outcomes cont. 3 7. Describe the morphology of the benign breast tumors with special reference to fibroadenoma and intraductal papilloma 8. List the risk factors of breast cancer 9. List the histological classification of breast cancer. 10.Explain the prognostic factors of the breast cancer with special reference to: stage at the time of diagnosis, histological features & Estrogen-receptor status 11.Describe the gross and microscopic features of different breast carcinoma with special reference to: in situ ductal carcinoma, in situ lobular carcinoma, invasive duct carcinoma, invasive lobular carcinoma & Paget’s disease of the breast.
  • 4. Marc Imhotep Cray, MD Learning Outcomes cont. 4 Female genital system: 1. Describe acute and chronic cervicitis. 2. Describe the pathogenesis, risk factors, clinical features and morphology of squamous cell carcinoma of cervix. 3. Describe the terms used in dysfunctional uterine bleeding and list its causes. 4. Describe endometritis with special references to various forms of endometritis. 5. Define endometriosis; explain the theories of endometriosis and its morphology and complications. 6. Describe endometrial polyps. FM Reproductive Anatomy_ The Noted Anatomist
  • 5. Marc Imhotep Cray, MD FM Genital System Learning Outcomes cont. 5 7. Describe the three types of endometrial hyperplasia. 8. Describe risk factors, etiology, clinical features, morphology and pathogenesis of endometrial carcinoma as well its morphology. 9. Describe the morphology and clinical features of leiomyomas of the uterus. 10.List the differences between leiomyoma and leiomyosarcoma of the uterus. 11.Classify tumors and tumor like lesions of the ovary 12.Describe the pathogenesis and morphology of surface epithelium tumors, teratomas and granulose theca cell tumors
  • 6. Marc Imhotep Cray, MD Female Breast Pathology 6
  • 7. Marc Imhotep Cray, MD Female Breast Functional Anatomy 7  Functional unit of breast is lobule supported by a specialized intralobular stroma  Inner luminal epithelial cells produce milk during lactation  Basally located myoepithelial cells have contractile function to aid in milk ejection  Ducts are conduits for milk to reach the nipple Size of breast is determined primarily by interlobular stroma increases during puberty and involutes with age Each normal constituent is a source of both benign and malignant lesions
  • 8. Marc Imhotep Cray, MD Normal breast, gross 8 Klatt EC. Robbins and Cotran Atlas of Pathology, 3rd Ed. 2015.
  • 9. Marc Imhotep Cray, MD Breast, mammogram 9  A mammogram uses a small amount of x-radiation to visualize the breast parenchyma  This mammogram shows normal pattern of lactiferous sinuses and ducts. There is one suspicious density ( ), however, which could be a carcinoma or just an area of pronounced sclerosis with fibrocystic changes  A mammogram is a useful screening tool to find such lesions and to determine need for further workup  A mammogram may detect lesions that are not palpable  Women in their 30s begin to have some involution of lobules and adjacent stroma, and breast tissue becomes more radiolucent from an increased composition of adipose tissue replacing fibrous stroma and lobules Klatt EC. Robbins and Cotran Atlas of Pathology, 3rd Ed. 2015.
  • 10. Marc Imhotep Cray, MD 10 Normal breast, microscopic Klatt EC. Robbins and Cotran Atlas of Pathology, 3rd Ed. 2015.
  • 11. Marc Imhotep Cray, MD 11 Origins of breast disorders Kumar, V; Abbas AK (Eds.), Robbins Basic Pathology, 10th Ed. Philadelphia: Elsevier, 2018; Fig.19.22, 736.  Benign epithelial lesions include intraductal papillomas that grow in sinuses below nipple & epithelial hyperplasia that arises in lobules  Malignant epithelial lesions are mainly breast carcinomas, which may remain in situ or invade into breast and spread by metastasis  Specialized intralobular stroma (green) cells may give rise to fibroadenomas and phyllodes tumors, whereas interlobular stroma (red) may give rise to a variety of rare benign and malignant tumors
  • 12. Marc Imhotep Cray, MD Clinical Presentations of Breast Disease 12 Predominant symptoms and signs of diseases of breast are pain, inflammatory changes, nipple discharge, “lumpiness,” or a palpable mass Kumar, V; Abbas AK (Eds.), Robbins Basic Pathology, 10th Ed. Philadelphia: Elsevier, 2018; Fig.19.23, 737.
  • 13. Marc Imhotep Cray, MD Risk Factors for Breast Cancer Development 13 Not Modifiable Modifiable Age Body mass index BRCA germline mutations Diet Family history Alcohol Chest radiation Exogenous estrogen Race/ethnicity Exercise Height Smoking Age at menarche Reproductive history Age at menopause Age at first full-term delivery Breast density Lactation Atypia on prior breast biopsy Redrawn after Rubin R and Strayer DS Eds. Rubin’s Essential of Pathology:, 6th Ed., 2014, Pg. 536.
  • 14. Marc Imhotep Cray, MD Clinical Presentations of Breast Disease: Key Points 14  Symptoms affecting breasts are evaluated primarily to determine if malignancy is present  Regardless of symptom, underlying cause is benign in majority of cases  Breast cancer is most commonly detected by palpation of a mass in younger women and in unscreened populations and by mammographic screening in older women
  • 15. Marc Imhotep Cray, MD Vignette #1 15 A 30-year-old woman presents to your clinic complaining of bilateral, diffuse breast pain. Her last menstrual period was 2 weeks ago and she says that she has felt several painful masses in both breasts during her self-examination in the shower. She is very concerned because she has a sister-in-law, who was recently diagnosed with breast cancer. On physical examination, you notice multiple, palpable masses on both breasts but no changes to the overlying skin. After taking a detailed history, you learn that the patient does not drink or smoke, has no immediate family members with a history of breast cancer, and there has been no nipple discharge. You order a fine needle aspiration cytology, which reveals an aspirate suggestive of a cyst. You reassure the patient that the lesion is benign and recommend that she avoid trauma to the affected regions and wear a brassiere that gives good support and protection. What is the Diagnosis?
  • 16. Marc Imhotep Cray, MD Fibrocystic Changes of the Breast 16 Etiology and Epidemiology: Caused by hormonal imbalance (increased estrogens and/or decreased progesterones); Peak incidence is between 25 and 50 years old Pathology: Four histologic types: 1. Cystic: multiple fluid-filled cysts appear blue (blue-dome cyst) cysts lined by polygonal cells with eosinophilic granular cytoplasm that are similar to apocrine epithelium (apocrine metaplasia), may see papillary projections of cystic epithelium; 2. Epithelial hyperplasia of breast duct: increase in number of epithelial layers in terminal duct lobules resulting in irregular lumens; 3. Stromal fibrosis: hyperplasia and fibrosis of breast stroma; 4. Sclerosing adenosis: increased number of acini, stromal fibrosis
  • 17. Marc Imhotep Cray, MD Fibrocystic Changes of the Breast cont. 17 Clinical Manifestations: Presents with diffuse breast pain (midcycle tenderness) and multiple, palpable lesions, often bilateral; no changes in overlying skin or nipple; rapid fluctuation in size of masses is common Treatment: Symptom management with pain control NB: Fibrocystic changes are most common breast disorder  Cystic and stromal fibrosis represent no increased risk for carcinoma, but  Epithelial hyperplasia and sclerosing adenosis do carry a mildly increased risk
  • 18. Marc Imhotep Cray, MD 18 Fibrocystic changes, gross Klatt EC. Robbins and Cotran Atlas of Pathology, 3rd Ed. 2015.
  • 19. Marc Imhotep Cray, MD 19 Fibrocystic changes, microscopic Klatt EC. Robbins and Cotran Atlas of Pathology, 3rd Ed. 2015.
  • 20. Marc Imhotep Cray, MD 20 Sclerosing adenosis, microscopic Klatt EC. Robbins and Cotran Atlas of Pathology, 3rd Ed. 2015.
  • 21. Marc Imhotep Cray, MD 21 Epithelial hyperplasia, microscopic Klatt EC. Robbins and Cotran Atlas of Pathology, 3rd Ed. 2015.
  • 22. Marc Imhotep Cray, MD 22 Normal breast, microscopic Klatt EC. Robbins and Cotran Atlas of Pathology, 3rd Ed. 2015.
  • 23. Marc Imhotep Cray, MD 23 Normal breast, microscopic Klatt EC. Robbins and Cotran Atlas of Pathology, 3rd Ed. 2015.
  • 24. Marc Imhotep Cray, MD 24 Normal breast, microscopic Klatt EC. Robbins and Cotran Atlas of Pathology, 3rd Ed. 2015.
  • 25. Marc Imhotep Cray, MD 25 Normal lactating breast, microscopic Klatt EC. Robbins and Cotran Atlas of Pathology, 3rd Ed. 2015.
  • 26. Marc Imhotep Cray, MD 26 Acute mastitis, microscopic Klatt EC. Robbins and Cotran Atlas of Pathology, 3rd Ed. 2015.
  • 27. Marc Imhotep Cray, MD 27 Breast abscess, gross Klatt EC. Robbins and Cotran Atlas of Pathology, 3rd Ed. 2015.
  • 28. Marc Imhotep Cray, MD Vignette #2 28 A 21-year-old African American woman presents to the clinic complaining of a large mass in her left breast. She has no immediate family members with breast cancer. On physical examination, you notice that the mass is round, rubbery, mobile, and nontender. It is approximately 4 cm in diameter. You order a needle biopsy that shows a combination of connective tissue and cystic spaces taking on a leaflike appearance. You inform the patient that this lesion is most often benign, but nevertheless you suggest that she be treated with a local excision to remove the growth. What is the Diagnosis?
  • 29. Marc Imhotep Cray, MD Benign Tumors of the Breast 29 Epidemiology: Fibroadenoma (FA): Occurs in women < 40; tends to occur more frequently and at a younger age in African American women; Phyllodes tumor (PT): Occurs most commonly after the age of 50; Intraductal papilloma (IP): Occurs in middle-aged women Pathology:  FA: Gross: small, mobile, rubbery, firm mass with sharp, well-circumscribed edges; Microscopic: fibroblastic stroma surrounding cystic and glandular spaces; may regress after menopause and demonstrate calcifications.  PT: Gross: large, bulky mass of connective tissue and cysts; Microscopic: cystic spaces on cut section of stroma contains leaflike projections from cyst walls; leaflike appearance on breast surface; 5%–10% undergoes malignant change with atypia (cystosarcoma phyllodes).  IP: Gross: arising from major lactiferous ducts; Microscopic: proliferation of ductal epithelial tissue in papillary growth manner; apocrine metaplasia
  • 30. Marc Imhotep Cray, MD Benign Tumors of the Breast cont. 30 Clinical Manifestations: FA and PT: Increased size and tenderness of mass with pregnancy or menstrual cycle; no overlying skin changes; no lymphadenopathy; no nipple retraction IP: Presents with nipple discharge Treatment: FA: No treatment or simple excision PT: Local excision with wide margin; can recur after resection IP: Simple excision Of note: Phyllodes tumor and intraductal papilloma carry a mildly increased risk of breast carcinoma
  • 31. Marc Imhotep Cray, MD 31 Fibroadenoma, gross Klatt EC. Robbins and Cotran Atlas of Pathology, 3rd Ed. 2015.
  • 32. Marc Imhotep Cray, MD 32 Fibroadenoma, microscopic Klatt EC. Robbins and Cotran Atlas of Pathology, 3rd Ed. 2015.
  • 33. Marc Imhotep Cray, MD 33 Phyllodes tumor, microscopic Klatt EC. Robbins and Cotran Atlas of Pathology, 3rd Ed. 2015.
  • 34. Marc Imhotep Cray, MD Phyllodes tumor, mammogram 34  This mammogram shows a bright, solid 10-cm rounded mass lesion consistent with a phyllodes tumor  It still has discrete margins, similar to a fibroadenoma, but is much larger  The biologic behavior of a phyllodes tumor is difficult to predict, and it may recur locally, but rarely are there high- grade lesions that can metastasize  Tend to occur at an older age than do fibroadenomas, most commonly in sixth decade Klatt EC. Robbins and Cotran Atlas of Pathology, 3rd Ed. 2015.
  • 35. Marc Imhotep Cray, MD 35 Intraductal papilloma, microscopic Klatt EC. Robbins and Cotran Atlas of Pathology, 3rd Ed. 2015.
  • 36. Marc Imhotep Cray, MD Vignette #3 36 A 50-year-old woman presents to your clinic after finding a mass on the upper outer quadrant of her left breast. After taking a thorough history, you learn that her mother died from breast cancer and her maternal aunt was also diagnosed with breast cancer at an early age. The patient started her period at age 11, did not bear any children, and has not been through menopause. On physical examination, she is markedly obese and you notice retraction of the skin and the nipple on her left breast. You locate the mass in question during your breast examination and find that it is fixed, hard, and nontender. The mass was not present on her last mammogram dating back 2 years. You also feel palpable axillary lymph nodes. You schedule the patient for an immediate mammography and needle biopsy to confirm your suspicions. What is the Diagnosis?
  • 37. Marc Imhotep Cray, MD Breast Carcinoma 37 Etiology and Epidemiology: Risk factors include:  Family history of first-degree relative with breast cancer at young age (highest risk),  Autosomal dominant inheritance of mutations in BRCA1 or BRCA2 gene,  female gender,  Increased age,  Early first menarche,  Delayed first pregnancy,  Nulliparity,  Late menopause,  Radiation exposure, and  exogenous estrogen use
  • 38. Marc Imhotep Cray, MD Breast Carcinoma Epidemiology cont. 38  Breast carcinoma is the most common malignancy of women globally (excluding nonmelanoma skin cancer) and causes majority of cancer deaths in women Incidence in United States decreased slightly in 2002 and then stabilized  changes attributed to a decrease in use of postmenopausal hormone therapy and a plateau in number of women undergoing mammographic screening  Worldwide incidence and mortality is increasing at an alarming rate  major factors underlying this trend in developing countries are thought to be social changes that increase breast cancer risk—specifically, delayed childbearing, fewer pregnancies, and reduced breastfeeding—combined with a lack of access to optimal health care
  • 39. Marc Imhotep Cray, MD Breast Carcinoma Pathology 39  Almost all breast malignancies are adenocarcinomas (>95%)  In the most clinically useful classification system, breast cancers are divided based on expression of hormone receptors:  estrogen receptor (ER) and progesterone receptor (PR)—and  expression of human epidermal growth factor receptor 2 (HER2, also known as ERBB2 & HER2/neu), into three major groups: • ER positive (HER2 negative; 50%–65% of cancers) • HER2 positive (ER positive or negative; 10%–20% of cancers) • Triple negative (ER, PR, and HER2 negative; 10%–20% of cancers) NB: These three groups show striking differences in patient characteristics, pathologic features, treatment response, metastatic patterns, time to relapse, and outcome
  • 40. Marc Imhotep Cray, MD Age and incidence of breast cancer subtypes 40  The three hormone receptors expression groups of the previous slide show striking differences in  patient characteristics,  pathologic features,  treatment response,  metastatic patterns,  time to relapse, and  outcome Kumar, V; Abbas AK (Eds.), Robbins Basic Pathology, 10th Ed. Philadelphia: Elsevier, 2018; Fig.19.26, 740.
  • 41. Marc Imhotep Cray, MD FM Breast Anatomy 41
  • 42. Marc Imhotep Cray, MD Breast Carcinoma Pathology cont. 42 1. Infiltrating ductal carcinoma: Tumor cells arranged in cords, islands, or glands embedded in dense fibrous stroma; may arise from ductal carcinoma in situ (DCIS) 2. Intraductal comedocarcinoma: Sheet of tumor cells confined within duct; central necrosis; periductal fibrosis with inflammation 3. Inflammatory: Lymphatic involvement of overlying skin 4. Paget disease: Paget cells (large cells w clear halo of pale cytoplasm) extend from ducts and invade epidermis of nipple; underlying ductal adenocarcinoma within subareolar excretory ducts always present
  • 43. Marc Imhotep Cray, MD Breast Carcinoma Pathology cont. 43 5. Infiltrating lobular: Often multiple and bilateral; cells line up (Indian file) with tumor cells surrounding lobule in target fashion; signet ring cells; may arise from lobular carcinoma in situ (LCIS) after many years 6. Medullary: Solid sheets of cells with large nucleoli in scant stroma; lymphocytic infiltrate 7. Mucinous (colloid): Pools of extracellular mucin surrounding tumor cell clusters; gelatinous consistency Note: Breast carcinoma is second most common cause of cancer death among women
  • 44. Marc Imhotep Cray, MD Breast Cancer Morphology 44  Most common location of tumors within breast is in upper outer quadrant (50%), followed by central portion (20%)  About 4% of women with breast cancer have bilateral primary tumors or sequential lesions in the same breast  Breast cancers are classified morphologically according to whether they have penetrated the basement membrane • Those that remain within this boundary are termed in situ carcinomas, and • those that have spread beyond it are designated invasive carcinomas
  • 45. Marc Imhotep Cray, MD Breast Cancer Morphology cont. 45 In classification, above main forms of breast carcinoma are as follows: A. Noninvasive 1. Ductal carcinoma in situ (DCIS) 2. Lobular carcinoma in situ (LCIS) B. Invasive 1. Invasive ductal carcinoma (includes all carcinomas that are not of a special type)-70% to 80% 2. Invasive lobular carcinoma- 10% to 15% 3. Carcinoma with medullary features-5% 4. Mucinous carcinoma (colloid carcinoma) -5% 5. Tubular carcinoma-5% 6. Other types Adapted from: Kumar, V; Abbas AK (Eds.), Robbins Basic Pathology, 10th Ed. Philadelphia, Pa : Elsevier, 2018;742.
  • 46. Marc Imhotep Cray, MD Noninvasive (in Situ) Carcinoma 46  Two morphologic types of noninvasive breast carcinoma: ductal carcinoma in situ (DCIS) and lobular carcinoma in situ (LCIS) “ terms ductal and lobular are misleading, as both types of CIS are thought to arise from cells in terminal duct that give rise to lobules*”  By definition, both "respect" the basement membrane and do not invade into stroma or lymphovascular channels • DCIS has a wide variety of histologic appearances, including solid, comedo, cribriform, papillary, micropapillary, and "clinging" types *From: Kumar, V; Abbas AK (Eds.), Robbins Basic Pathology, 10th Ed. Philadelphia: Elsevier, 2018;743.
  • 47. Marc Imhotep Cray, MD Carcinoma in situ. (A) Lobular carcinoma in situ (LCIS). (B) Ductal carcinoma in situ (DCIS). DCIS partially involves the lobule in the lower half of this photo and has completely effaced the upper lobules, producing a ductlike appearance. (C) Mammographic detection of calcifications associated with DCIS. 47 Kumar, V; Abbas AK (Eds.), Robbins Basic Pathology, 10th Ed. Philadelphia: Elsevier, 2018; Fig.19.28,743.
  • 48. Marc Imhotep Cray, MD Vignette #4 48 A 49-year-old woman presents to your office concerned about a rash on her left nipple that has developed over the past month. The rash is itchy, but painless. On physical examination, you find a large eczematous-like patch over the left nipple as well as a fixed mass in the left breast. You perform both a skin biopsy as well as a needle biopsy of the mass. When the skin biopsy reveals large cells with a clear halo of pale cytoplasm invading the epidermis, you become certain that the biopsy of the mass will reveal carcinoma of the breast.
  • 49. Marc Imhotep Cray, MD Breast Carcinoma cont. 49 Clinical Manifestations: Painless, usually fixed, hard, nontender mass often found in upper outer quadrant of breast; retraction of overlying skin and nipple; palpable axillary lymph nodes  Infiltrating ductal carcinoma: Firm, fixed, fibrous mass  Inflammatory: Red, swollen, hot, painful to touch, orange-peel appearance of skin  Paget disease: Itchy, scaly, painless, eczematous patches on nipple  Medullary: Soft, fleshy-consistency mass Lab findings: Paraneoplastic syndrome with secretion of PTH-related peptide may lead to hypercalcemia Imaging: Mammogram with microcalcifications, spiculated or enlarging mass
  • 50. Marc Imhotep Cray, MD Breast Carcinoma cont. 50 Treatment:  Surgery and radiation therapy;  Chemotherapy;  Hormonal therapy (tamoxifen or aromatase inhibitors) for patients with cancer cells expressing estrogen receptor in their nuclei (ER/PR+);  Biologic therapy Trastuzumab (herceptin) for patients with HER2/neu expression NB: Metastasis occurs to lymph nodes, lung, liver, and bone.
  • 51. Marc Imhotep Cray, MD 51 Ductal carcinoma in situ, microscopic Klatt EC. Robbins and Cotran Atlas of Pathology, 3rd Ed. 2015.
  • 52. Marc Imhotep Cray, MD 52 Lobular carcinoma in situ, microscopic Klatt EC. Robbins and Cotran Atlas of Pathology, 3rd Ed. 2015.
  • 53. Marc Imhotep Cray, MD 53 Paget disease of breast, microscopic Klatt EC. Robbins and Cotran Atlas of Pathology, 3rd Ed. 2015.
  • 54. Marc Imhotep Cray, MD 54 Invasive ductal carcinoma, microscopic Klatt EC. Robbins and Cotran Atlas of Pathology, 3rd Ed. 2015.
  • 55. Marc Imhotep Cray, MD 55 Invasive ductal carcinoma, gross and mammogram Klatt EC. Robbins and Cotran Atlas of Pathology, 3rd Ed. 2015.
  • 56. Marc Imhotep Cray, MD 56 Infiltrating ductal carcinoma, gross and mammogram Klatt EC. Robbins and Cotran Atlas of Pathology, 3rd Ed. 2015.
  • 57. Marc Imhotep Cray, MD Breast Carcinoma Key Points Summary 57  The lifetime risk of developing breast cancer for an American woman is 1 in 8.  A majority (75%) of breast cancers are diagnosed after the age of 50.  The major risk factors for developing breast cancer are related to hormonal factors and inherited susceptibility.  About 12% of all breast cancers are caused by identified germline mutations; BRCAI and BRCA2 genes account for one-half of the cases associated with single-gene mutations.  DCIS is a precursor to invasive ductal carcinoma and is most often found on mammographic screening as calcifications.  When carcinoma develops in a woman with a previous diagnosis of untreated DCIS, it is usually is an invasive ductal carcinoma in the same breast.
  • 58. Marc Imhotep Cray, MD 58  LCIS is both a marker of increased risk and a precursor lesion.  When carcinoma develops in a woman with a previous diagnosis of LCIS, two- thirds are in the same breast and one-third is in the contralateral breast.  Invasive carcinomas are classified according to histologic type and biologic type: ER-positive/HER2-negative, HER2-positive, and ER/PR/HER2-negative (triple-negative). The biologic types of cancer have important differences in patient characteristics, grade, mutation profile, metastatic pattern, response to therapy, time to recurrence, and prognosis.  Prognosis is dependent on the biologic type of tumor, stage, and the availability of treatment modalities. Breast Carcinoma Key Points Summary (2)
  • 59. Marc Imhotep Cray, MD Ovaries and Uterus Pathology 59
  • 60. Marc Imhotep Cray, MD Normal internal genitalia, gross 60 Klatt EC. Robbins and Cotran Atlas of Pathology, 3rd Ed. 2015.
  • 61. Marc Imhotep Cray, MD Normal internal genitalia, radiograph 61 Klatt EC. Robbins and Cotran Atlas of Pathology, 3rd Ed. 2015.
  • 62. Marc Imhotep Cray, MD Normal fallopian tube, microscopic 62 Klatt EC. Robbins and Cotran Atlas of Pathology, 3rd Ed. 2015.
  • 63. Marc Imhotep Cray, MD Normal adult ovary, microscopic 63 Klatt EC. Robbins and Cotran Atlas of Pathology, 3rd Ed. 2015.
  • 64. Marc Imhotep Cray, MD Normal ovary, microscopic 64 Klatt EC. Robbins and Cotran Atlas of Pathology, 3rd Ed. 2015.
  • 65. Marc Imhotep Cray, MD Corpus luteum, gross 65 Klatt EC. Robbins and Cotran Atlas of Pathology, 3rd Ed. 2015.
  • 66. Marc Imhotep Cray, MD Menstrual cycle capsular summary:  Menstrual cycle is divided into follicular phase and luteal phase Ovulation defines transition between these two phases  During follicular phase, gonadotroph cells of anterior pituitary gland secrete LH and FSH in response to pulsatile GnRH stimulation  Circulating LH and FSH promote growth and maturation of ovarian follicles  Developing follicles secrete increasing amounts of estrogen  At first, estrogen has an inhibitory effect on gonadotropin release Just before midpoint in menstrual cycle, however, estrogen exerts a brief positive feedback effect on LH (LH Surge) and FSH release This is followed by follicular rupture and release of an egg into fallopian tube  During second half of cycle, corpus luteum secretes both estrogen and progesterone  Progesterone induces a change in endometrium from a proliferative to a secretory type  If fertilization and implantation of a blastocyst do not occur within 14 days after ovulation corpus luteum involutes secretion of estrogen and progesterone declines, menses occurs, and a new cycle begins Cairo CW, Simon JB, Golan DE. (Eds.). Principles of Pharmacology: The Pathophysiologic Basis of Drug Therapy. LLW, 2012.
  • 67. Marc Imhotep Cray, MD Hormonal Control of the Menstrual Cycle, Animation. 67 Online Version
  • 68. Marc Imhotep Cray, MD Menstruation terminology 68  Dysmenorrhea= Pain with menses often associated with endometriosis  Oligomenorrhea > 35-day cycle  Polymenorrhea < 21-day cycle  Metrorrhagia= Frequent or irregular menstruation  Menorrhagia= Heavy menstrual bleeding > 80 mL blood loss or > 7 days of menses  Menometrorrhagia= Heavy, irregular menstruation
  • 69. Marc Imhotep Cray, MD Vignette #5 69 A 45-year-old woman presents to the clinic complaining of vague abdominal pain for the past 4 days. She describes her pain as more of a pelvic pressure that is generalized bilaterally. Her last menstrual period was 2 weeks ago and she states that she has regular menstrual cycles. She denies the possibility of pregnancy and is currently not taking oral contraceptives. She has not had any changes in digestive functions and denies any nausea, vomiting, constipation, or diarrhea. She has no family history of ovarian cancer. When an ultrasound confirms your suspicions, you place the patient on oral contraceptive pills, believing that her symptoms will disappear in 2 months, and you schedule her for a follow-up ultrasound. What is the Diagnosis?
  • 70. Marc Imhotep Cray, MD Ovarian Cysts 70 Etiology and Epidemiology:  Follicular (F) cyst: Associated with hyperestrinism and endometrial hyperplasia; most common cause of ovarian enlargement; mostly found during menstrual years  Corpus luteum (CL) cyst: Found during menstrual years  Theca-lutein (TL) cyst: Associated with choriocarcinoma, hydatidiform moles, and clomiphene (synthetic gonadotropin) therapy Pathology  F: Often bilateral; distention of unruptured Graafian follicle; lined by granulosa cells  CL: Often unilateral; contains clear fluid; lined by yellowish luteal cells with cytoplasmic lipid droplets; may hemorrhage into persistent mature corpus luteum  TL: Often bilateral and multiple; lined by luteinized theca cells
  • 71. Marc Imhotep Cray, MD Ovarian Cysts cont. 71 Clinical Manifestations: All may be asymptomatic or present with pelvic pressure/pain or vague GI discomfort  F: Pain not associated with menstruation  CL: Delayed menstruation  TL: Amenorrhea Lab findings: hCG elevated as a result of trophoblastic proliferation Treatment:  F: Often disappears with 2-month regimen of oral contraceptives; follow with serial ultrasounds; laparoscopic removal if persistent  CL and TL: Cyst removal or unilateral oophorectomy
  • 72. Marc Imhotep Cray, MD Vignette #6 72 A 30-year-old white woman presents to the fertility clinic with her husband, complaining of the inability to conceive. The couple has already checked the husband’s sperm motility and count, which are within the normal range. The patient informs you that she has not menstruated for the last 4 months and that she has had a very irregular and sporadic menstrual cycle all of her life. You perform a physical examination, finding that the patient is obese with an inordinate amount of facial hair. She denies any abnormal uterine bleeding. You order serum studies that show elevated plasma LH and testosterone and decreased FSH levels. Based on these findings, you inform the patient that weight reduction will be the most effective treatment for restoring ovulation and that adjunct therapy with clomiphene can aid in ovulation. What is the Diagnosis?
  • 73. Marc Imhotep Cray, MD Polycystic Ovarian Syndrome (Stein-Leventhal Syndrome) 73 Etiology and Epidemiology: Etiology unclear, but it is believed that a dysregulation of enzymes involved in androgen biosynthesis may be caused by increased LH secretion thereby resulting in excessive production of androgens; associated with obesity, Cushing syndrome, congenital adrenal hyperplasia, genetic predisposition, and androgen-secreting adrenal tumors Common endocrine disorder affecting 2%–5% of women during reproductive age Pathology and Pathophysiology: Pathophysiology: Increased androgen production causes anovulation, multiple follicular cysts, and theca cell hyperplasia Gross: Ovaries enlarged; pearly white thickened ovarian capsule; multiple cysts Microscopic: Cysts have granulosa cell layer and luteinized theca cells; cortical stromal fibrosis
  • 74. Marc Imhotep Cray, MD PCOS cont. 74 Clinical Manifestations: Amenorrhea; infertility; obesity; hirsutism (in 70%); insulin resistance with increased risk of diabetes; virilism; increased risk of breast and endometrial carcinoma Lab findings: Increased LH, decreased FSH, increased testosterone, evidence of insulin resistance Treatment: Weight loss; oral contraceptives; metformin for insulin resistance; gonadotropin analogs; ovulation induction with clomiphene
  • 75. Marc Imhotep Cray, MD Polycystic ovary, MRI 75 Klatt EC. Robbins and Cotran Atlas of Pathology, 3rd Ed. 2015.
  • 76. Marc Imhotep Cray, MD Polycystic ovarian syndrome, microscopic 76 Klatt EC. Robbins and Cotran Atlas of Pathology, 3rd Ed. 2015.
  • 77. Marc Imhotep Cray, MD Vignette #7 77 A 40-year-old woman presents to the emergency room with generalized abdominal pain and pelvic pressure. After taking a more complete history, you learn that she has not passed stool for the last 3 days and vomited this morning. She tells you that the pain has been steady over the last week. You order an abdominal/pelvic CT scan, which is inconclusive, but does demonstrate bilateral enlargement of the ovaries. The patient is taken to the operating room for an exploratory laparotomy that shows extensive mucinous ascites, cystic epithelial implants on the peritoneal surfaces, and several adhesions. You believe that the patient’s mucinous peritoneal involvement is caused by her ovarian condition. What is the Diagnosis?
  • 78. Marc Imhotep Cray, MD Ovarian Tumors of Surface Epithelium Origin 78 Epidemiology: Usually occurs in women > 20 years of age Pathology:  Serous cystadenoma: Bilateral, benign cyst with fallopian tube-like epithelium  Papillary serous cystadenocarcinoma: Bilateral, malignant cysts lined by stratified atypical epithelium; papillary growth; psammoma bodies  Mucinous cystadenoma: Multilocular, benign cysts with columnar cells filled with mucin  Mucinous cystadenocarcinoma: Malignant tumor with mucus-secreting atypical columnar epithelium; loss of gland architecture; necrosis  Brenner tumor: Benign tumor with nests of cells resembling bladder transitional epithelium interspersed in fibrous stroma  Endometrioid tumor: Malignant tumor resembling endometrium Clear cell tumor: Rare, malignant tumor composed of sheets of clear cells
  • 79. Marc Imhotep Cray, MD Ovarian Tumors of Surface Epithelium Origin cont. 79 Clinical Manifestations: Mild, nonspecific abdominal discomfort; malignant forms may present with weakness, weight loss, and anorexia; pseudomyxoma peritonei (intraperitoneal accumulation of mucinous material) is associated with mucinous cystadenocarcinoma Lab finding: Elevated CA-125 in ovarian carcinomas Treatment: Tumor removal; oophorectomy or hysterectomy; chemotherapy Of note: Ovarian epithelial tumors make up 75% of ovarian tumors.
  • 80. Marc Imhotep Cray, MD Serous cystadenoma, gross 80 Klatt EC. Robbins and Cotran Atlas of Pathology, 3rd Ed. 2015.
  • 81. Marc Imhotep Cray, MD 81 Serous cystadenoma, MRI and CT image Klatt EC. Robbins and Cotran Atlas of Pathology, 3rd Ed. 2015.
  • 82. Marc Imhotep Cray, MD Multiloculated ovarian tumor, gross 82 Klatt EC. Robbins and Cotran Atlas of Pathology, 3rd Ed. 2015.
  • 83. Marc Imhotep Cray, MD Cystadenoma, serous, mucinous, microscopic 83 Klatt EC. Robbins and Cotran Atlas of Pathology, 3rd Ed. 2015.
  • 84. Marc Imhotep Cray, MD Borderline serous tumor, microscopic 84 Klatt EC. Robbins and Cotran Atlas of Pathology, 3rd Ed. 2015.
  • 85. Marc Imhotep Cray, MD Borderline tumor, gross 85 Klatt EC. Robbins and Cotran Atlas of Pathology, 3rd Ed. 2015.
  • 86. Marc Imhotep Cray, MD Cystadenocarcinoma & peritoneal metastases, CT images 86 Klatt EC. Robbins and Cotran Atlas of Pathology, 3rd Ed. 2015.
  • 87. Marc Imhotep Cray, MD Cystadenocarcinoma, gross 87 Klatt EC. Robbins and Cotran Atlas of Pathology, 3rd Ed. 2015.
  • 88. Marc Imhotep Cray, MD Cystadenocarcinoma, microscopic 88 Klatt EC. Robbins and Cotran Atlas of Pathology, 3rd Ed. 2015.
  • 89. Marc Imhotep Cray, MD Endometrioid tumor, microscopic 89 Klatt EC. Robbins and Cotran Atlas of Pathology, 3rd Ed. 2015.
  • 90. Marc Imhotep Cray, MD Brenner tumor, microscopic 90 Klatt EC. Robbins and Cotran Atlas of Pathology, 3rd Ed. 2015.
  • 91. Marc Imhotep Cray, MD Vignette #8 91 An 18-year-old white woman is admitted to your gynecologic service with a 1- month history of increasing abdominal pain and a rapidly enlarging pelvic mass. While admitting the patient, you perform a bimanual pelvic examination that reveals a left ovarian mass. This finding is consistent with the admitting note and was confirmed by abdominal/pelvic CT, which showed that the left ovary was twice as large as the right ovary. Serum studies demonstrate an elevated AFP level. There is no inguinal lymph node involvement and no signs of metastasis on imaging studies. Based on your findings, you perform a unilateral oophorectomy and send the diseased ovary for pathologic analysis. Histology shows glomerulus-like structures composed of a central blood vessel enveloped by germ cells within a space similarly lined by germ cells. These histologic bodies confirm your suspected diagnosis and you start the patient on combination chemotherapy What is the Diagnosis?
  • 92. Marc Imhotep Cray, MD Ovarian Tumors of Germ Cell Origin 92 Etiology and Epidemiology: Risk factors include nulliparity, positive family history of ovarian cancer, mutations in BRCA1 and BRCA2 genes and high expression of HER2/neu oncogene; Occurs most commonly in children and young adults (except for teratomas, which can occur at all ages) Pathology:  Dysgerminoma: Malignant unilateral tumor comprised of large vesicular cells with clear cytoplasm and central nuclei; analogous to male testicular seminoma  Yolk sac tumor: Malignant tumor with Schiller-Duval bodies (glomerulus-like structure composed of central blood vessel enveloped by germ cells)  Choriocarcinoma: Aggressive and malignant tumor with areas of necrosis and hemorrhage composed of neoplastic syncytiotrophoblasts and cytotrophoblasts  Teratoma: 90% of germ cell tumors; mature teratomas (dermoid cysts) are benign; immature are malignant ; Histology: structures from all germ layers.  Struma ovarii: Unilateral ovarian teratoma composed of thyroid tissue
  • 93. Marc Imhotep Cray, MD Ovarian Tumors of Germ Cell Origin cont. 93 Clinical Manifestations: Mild, nonspecific abdominal discomfort; struma ovarii may lead to hyperthyroidism Lab findings: Increased AFP (yolk sac), increased hCG (choriocarcinoma) Tx: Tumor removal; oophorectomy or hysterectomy; chemotherapy Note: Germ cell tumors make up 25% of ovarian tumors
  • 94. Marc Imhotep Cray, MD 94 Mature cystic teratoma, gross Klatt EC. Robbins and Cotran Atlas of Pathology, 3rd Ed. 2015.
  • 95. Marc Imhotep Cray, MD 95 Mature cystic teratoma, CT image Klatt EC. Robbins and Cotran Atlas of Pathology, 3rd Ed. 2015.
  • 96. Marc Imhotep Cray, MD 96 Mature cystic teratoma, microscopic Klatt EC. Robbins and Cotran Atlas of Pathology, 3rd Ed. 2015.
  • 97. Marc Imhotep Cray, MD 97 Dysgerminoma, gross Klatt EC. Robbins and Cotran Atlas of Pathology, 3rd Ed. 2015.
  • 98. Marc Imhotep Cray, MD 98 Dysgerminoma, microscopic Klatt EC. Robbins and Cotran Atlas of Pathology, 3rd Ed. 2015.
  • 99. Marc Imhotep Cray, MD Vignette #9 99 A 12-year-old girl is brought into the emergency room because of heavy vaginal bleeding. The child is also complaining of GI discomfort and pelvic pressure. She states that she had her last menstruation 2 weeks earlier and she is not sexually active. Her first menstruation was at the age of 10. On physical examination, the child appears to have undergone precocious puberty. A pelvic examination reveals a palpable right ovarian mass, which is confirmed by a CT scan. Serum studies show a significantly increased level of circulating estrogen. When a biopsy reveals distinctive, gland-like structures filled with an acidophilic material, you worry that this patient may have an increased risk of developing endometrial carcinoma later in life if this mass is not removed. What is the Diagnosis?
  • 100. Marc Imhotep Cray, MD Ovarian Tumors of Sex Cord–Stromal Origin 100 Etiology and Epidemiology: Risk factors include nulliparity, positive family history of ovarian cancer, mutations in BRCA1 and BRCA2 genes, and high expression of HER2/neu oncogene; Affects all age groups Pathology and Pathophysiology:  Ovarian fibroma-thecoma (OFT): Secretes estrogen; tumor composed of round lipid-containing cells in addition to well-differentiated fibroblasts  Granulosa cell tumor (GCT): Secretes estrogen causing endometrial hyperplasia/carcinoma in adults; characterized by Call-Exner bodies (small follicles filled with eosinophilic secretions) and small cuboidal, deeply stained granulosa cells arranged in anastomotic cords  Sertoli-Leydig cell tumor (SLCT): Secretes androgens; tumor composed of Sertoli or Leydig cells interspersed with stroma
  • 101. Marc Imhotep Cray, MD Ovarian Tumors of Sex Cord–Stromal Origin cont. 101 Clinical Manifestations: Mild, nonspecific abdominal discomfort  OFT: Presents with Meig syndrome (triad of ovarian tumor, ascites, and hydrothorax)  GCT: Vaginal bleeding from endometrial hyperplasia; cystic disease of breast; endometrial carcinoma  SLCT: Virilism Lab findings: Increased estrogen levels (OFT and GCT) Treatment: Tumor removal; oophorectomy or hysterectomy; chemotherapy Note: Ovarian sex cord–stromal tumors are rare
  • 102. Marc Imhotep Cray, MD 102 Granulosa–theca cell tumor, gross Klatt EC. Robbins and Cotran Atlas of Pathology, 3rd Ed. 2015.
  • 103. Marc Imhotep Cray, MD 103 Granulosa cell tumor, microscopic Klatt EC. Robbins and Cotran Atlas of Pathology, 3rd Ed. 2015.
  • 104. Marc Imhotep Cray, MD 104 Thecoma-fibroma, gross Klatt EC. Robbins and Cotran Atlas of Pathology, 3rd Ed. 2015.
  • 105. Marc Imhotep Cray, MD 105 Thecoma-fibroma, microscopic Klatt EC. Robbins and Cotran Atlas of Pathology, 3rd Ed. 2015.
  • 106. Marc Imhotep Cray, MD Vignette #10 106 A 31-year-old woman presents to the clinic complaining of abnormal vaginal bleeding. She states that her last menstrual period was 1 week ago and that she has not experienced abnormal bleeding before. Her past medical history is notable for two prior episodes of pelvic inflammatory disease in the last year. She has also been trying to conceive for the last year without success. On pelvic examination, redness and inflammation of the cervix is visible, but no discharge is apparent. You obtain an endometrial biopsy, which you suspect will show plasma cells along with macrophages and leukocytes in the glandular lumen. You begin empiric antibiotic therapy to prevent possible sequelae from the suspected diagnosis. What is the Diagnosis?
  • 107. Marc Imhotep Cray, MD Endometritis 107 Etiology:  Acute endometritis: Caused by trauma, Staphylococcus aureus and Streptococcus species; usually occurs after delivery or miscarriage;  Chronic endometritis: Caused by granulomatous disease, chronic PID, postpartal or postabortal states, and TB Pathology: Endometrium: Plasma cells seen with macrophages and lymphocytes Clinical Manifestations: Acute: Presents with inflammation after delivery or miscarriage Chronic: Presents with abnormal vaginal bleeding, pain, discharge, and infertility Treatment: Antibiotic Tx (helps to prevent other sequelae, such as salpingitis) Of Note: Endometrial polyps are masses composed of endometrial tissue within endometrial cavity  occur in women older than 40 years of age and may result in uterine bleeding, but are usually benign
  • 108. Marc Imhotep Cray, MD 108 Acute endometritis, microscopic Klatt EC. Robbins and Cotran Atlas of Pathology, 3rd Ed. 2015.
  • 109. Marc Imhotep Cray, MD 109 Chronic endometritis, microscopic Klatt EC. Robbins and Cotran Atlas of Pathology, 3rd Ed. 2015.
  • 110. Marc Imhotep Cray, MD 110 Granulomatous endometritis, microscopic Klatt EC. Robbins and Cotran Atlas of Pathology, 3rd Ed. 2015.
  • 111. Marc Imhotep Cray, MD Vignette #11 111 A 24-year-old woman presents to the clinic complaining of increased pain and bleeding during menstruation. Her last three menstruations have been accompanied by increasing intensity of cramping and larger amounts of blood. She tells you that her menstrual cycle has been irregular for the last 6 months. After taking a complete history, you learn that she has been having increased pelvic pain with intercourse. On pelvic examination, you palpate fixed, bilateral ovarian masses and an MRI reveals chocolate cysts on the ovary. You begin the patient on oral contraceptives and you suggest surgical removal of the masses if the pain persists. What is the Diagnosis?
  • 112. Marc Imhotep Cray, MD Endometriosis 112 Etiology and Epidemiology: Etiology unknown, but thought to be caused by a combination of genetic, hormonal, and immune factors; Afflicts 10% of women, usually between the ages of 20 and 30 Pathology:  Gross: Non-neoplastic nodules, composed of endometrial tissue in abnormal locations outside the uterus, including ovary (most common), uterine ligaments, rectovaginal septum, and pelvic peritoneum; presence of chocolate cysts on ovaries that have resulted from cyclic bleeding (menstrual type) of ectopic endometrial tissue  Microscopic: Endometrial glands; endometrial stroma; presence of hemosiderin pigment
  • 113. Marc Imhotep Cray, MD Endometriosis cont. 113 Clinical Manifestations: Presents with fixed, palpable, bilateral ovarian masses, pain during menses (dysmenorrhea), and pain during intercourse (dyspareunia); menstrual irregularities may lead to infertility presenting complaint of 30%–40% of pts. Treatment: Oral contraceptives; progesterone; danazol; GnRH; surgical removal/coagulation (cauterization) of lesion Note: Adenomyosis is endometriosis within the myometrium and usually presents with uterine enlargement and irregular bleeding
  • 114. Marc Imhotep Cray, MD 114 Adenomyosis, MRI Klatt EC. Robbins and Cotran Atlas of Pathology, 3rd Ed. 2015.
  • 115. Marc Imhotep Cray, MD 115 Adenomyosis, gross Klatt EC. Robbins and Cotran Atlas of Pathology, 3rd Ed. 2015.
  • 116. Marc Imhotep Cray, MD 116 Adenomyosis, microscopic Klatt EC. Robbins and Cotran Atlas of Pathology, 3rd Ed. 2015.
  • 117. Marc Imhotep Cray, MD 117 Endometriosis, gross Klatt EC. Robbins and Cotran Atlas of Pathology, 3rd Ed. 2015.
  • 118. Marc Imhotep Cray, MD 118 Endometriosis, gross
  • 119. Marc Imhotep Cray, MD 119 Endometriosis, microscopic Klatt EC. Robbins and Cotran Atlas of Pathology, 3rd Ed. 2015.
  • 120. Marc Imhotep Cray, MD 120 Endometrial polyp, gross Klatt EC. Robbins and Cotran Atlas of Pathology, 3rd Ed. 2015.
  • 121. Marc Imhotep Cray, MD Vignette #12 121 A 35-year-old African American woman presents to the clinic complaining of more frequent menstrual periods and profuse menstruation. She also complains of increased weakness and fatigue during her menstruations. On physical examination, you find a firm abdominal mass in the pelvic region. When questioned, she states that she was aware of the mass, but she notes that the mass often only appears during menstrual periods. When a biopsy of the mass reveals whorled bundles of smooth muscle cells, you reassure this patient that her condition is unlikely to proceed to malignancy. What is the Diagnosis?
  • 122. Marc Imhotep Cray, MD Leiomyoma (fibroids) 122 Etiology and Epidemiology: Etiology unknown, but chromosomal abnormalities have been found in tumor cells; Most common benign neoplasm of female genital tract; Most commonly seen in women of reproductive age with an increased incidence in African Americans Pathology: Gross: Multiple, enlarged, irregular, heterogenous tumors in the myometrium (intramural), beneath endometrium (submucosal), or beneath serosa (subserosal); tumor is estrogen-sensitive with its size increasing during pregnancy and decreasing with menopause Microscopic: Whorled pattern of smooth muscle bundles; rare mitoses in muscle cells, although cellular atypia and giant cells may be present
  • 123. Marc Imhotep Cray, MD Leiomyoma (fibroids) cont. 123 Clinical Manifestations: Profuse menstruation; frequent menstrual periods; acute or recurrent pelvic pain; urinary frequency (owing to compression of bladder); infertility Lab findings: Iron deficiency anemia (owing to blood loss) Treatment: Myomectomy or hysterectomy; uterine artery embolization Note: Also called uterine fibroids, leiomyomas do not progress to leiomyosarcomas  Leiomyosarcomas are fleshy, irregular tumors with areas of necrosis and hemorrhage that arise de novo and may protrude from cervix  Leiomyosarcomas are more prevalent among African Americans, are malignant, and can be treated with combination chemotherapy
  • 124. Marc Imhotep Cray, MD 124 Leiomyomata, gross Klatt EC. Robbins and Cotran Atlas of Pathology, 3rd Ed. 2015.
  • 125. Marc Imhotep Cray, MD Uterine fibroids as seen during laparoscopic surgery 125 https://commons.wikimedia.org/wiki/File:Uterine_fibroids.jpg
  • 126. Marc Imhotep Cray, MD 126 Leiomyomata, MRI and CT image Klatt EC. Robbins and Cotran Atlas of Pathology, 3rd Ed. 2015.
  • 127. Marc Imhotep Cray, MD 127 Leiomyoma, microscopic Klatt EC. Robbins and Cotran Atlas of Pathology, 3rd Ed. 2015.
  • 128. Marc Imhotep Cray, MD 128 Leiomyosarcoma, gross Klatt EC. Robbins and Cotran Atlas of Pathology, 3rd Ed. 2015.
  • 129. Marc Imhotep Cray, MD 129 Leiomyosarcoma, microscopic Klatt EC. Robbins and Cotran Atlas of Pathology, 3rd Ed. 2015.
  • 130. Marc Imhotep Cray, MD Vignette #13 130 A 60-year-old woman presents to the clinic with postmenopausal vaginal bleeding. After taking a complete history, you learn that she is nulliparous and suffers from type 2 diabetes, which is well-controlled with diet and insulin. On physical examination, the woman is obese and has a blood pressure reading of 150/96. You decide to perform a PAP smear as well as an endometrial biopsy. Based on the patient’s presenting signs and medical history, you are worried that you might find well-defined gland patterns lined by malignant stratified columnar epithelial cells on endometrial biopsy. What is the Diagnosis?
  • 131. Marc Imhotep Cray, MD Endometrial Carcinoma 131 Etiology and Epidemiology: Risk factors include unopposed estrogen use, obesity, diabetes, HTN, nulliparity, and late menopause Peak incidence is between 55 and 65 years of age Endometrial carcinoma is most common gynecologic malignancy Pathology: Typically preceded by endometrial hyperplasia Gross: Localized polypoid tumor or diffuse tumor involving entire endometrial surface Microscopic: Adenocarcinoma characterized by well-defined gland patterns lined by malignant stratified columnar epithelial cells; may see some squamous cells
  • 132. Marc Imhotep Cray, MD Endometrial Ca. cont. 132 Clinical Manifestations: Presents with postmenopausal vaginal bleeding, leading to early diagnosis; may cause obstruction of cervix with collection of pus (pyometra) or blood (hematometra) presenting with lower abdominal pain Treatment: Total hysterectomy and bilateral salpingo-oophorectomy; radiation therapy Important Note:  Endometrial hyperplasia is abnormal endometrial gland proliferation caused by excess estrogen (eg, polycystic ovarian syndrome, estrogen-secreting ovarian tumor, estrogen replacement therapy)  It manifests clinically with postmenopausal vaginal bleeding  may lead to endometrial cancer depending on degree of atypia
  • 133. Marc Imhotep Cray, MD 133 Endometrial atypical hyperplasia, microscopic Klatt EC. Robbins and Cotran Atlas of Pathology, 3rd Ed. 2015.
  • 134. Marc Imhotep Cray, MD 134 Endometrial carcinoma, gross Klatt EC. Robbins and Cotran Atlas of Pathology, 3rd Ed. 2015.
  • 135. Marc Imhotep Cray, MD 135 Endometrial carcinoma, gross (2) Klatt EC. Robbins and Cotran Atlas of Pathology, 3rd Ed. 2015.
  • 136. Marc Imhotep Cray, MD 136 Endometrial carcinoma, type I, microscopic Klatt EC. Robbins and Cotran Atlas of Pathology, 3rd Ed. 2015.
  • 137. Marc Imhotep Cray, MD 137 Endometrial carcinoma, type II, microscopic Klatt EC. Robbins and Cotran Atlas of Pathology, 3rd Ed. 2015.
  • 138. Marc Imhotep Cray, MD Vignette #14 138 A 42-year-old woman presents to the clinic complaining of postcoital bleeding and vaginal discharge. She complains of a 3-month history of spotting in her underwear after intercourse and an odorous vaginal discharge that is not purulent. Her social history is significant for past practice of prostitution and her past medical history is significant for several STDs that were appropriately treated. She has not had a routine PAP smear in over 10 years. After a PAP smear reveals abnormal cells, you perform a cervical biopsy, worrying that you may find invasive malignant cells in the cervix and adjacent koilocytosis. You fear that the patient will need to undergo a hysterectomy with possible adjunct radiation therapy if your suspected diagnosis is confirmed. What is the Diagnosis?
  • 139. Marc Imhotep Cray, MD Dysplasia, Carcinoma in Situ, and Squamous Cell Carcinoma of the Cervix 139 Etiology and Epidemiology: Associated with human papilloma virus (HPV) types 16, 18, 31, and 33, early age of first intercourse and multiple sexual partners Occurs most commonly between the ages of 30 and 45 Pathology: Cervical dysplasia: Involves squamocolumnar junction; characterized by cells with hyperchromatic nuclei, irregular nuclear contours, and scant cytoplasm; epithelial growth begins at basal layer extending outward; classified as Cervical Intraepithelial Neoplasia (CIN) Subtypes Grades I-III (or Squamous Intraepithelial Lesions Low-grade to High-grade )  CIN I (LSIL) is characterized by atypical undifferentiated cells only in lower third of epithelium, whereas  CIN II (HSIL) superficial layer of cells still shows differentiation and koilocytosis  CIN III (HSIL) has atypia through > 2/3 thickness of epithelium, koilocytotic change usually is absent
  • 140. Marc Imhotep Cray, MD Dysplasia, Carcinoma in Situ, and Squamous Cell Carcinoma of the Cervix (2) 140 Pathology cont: Cervical carcinoma in situ (CIS): Dysplastic cells extending through entire epithelium, but without invasion of basement membrane Invasive cervical carcinoma (ICC): Gross: can be exophytic, ulcerating, or infiltrating mass Microscopic: usually squamous cell carcinoma with large cells and keratinization; can be adenocarcinoma or undifferentiated carcinoma; arises from preexisting CIN at squamocolumnar junction; non-neoplastic epithelial cells often demonstrate koilocytosis (assoc. w. HPV infection) Koilocytes, also known as halo cells, are a type of epithelial cell that develops following a human papillomavirus (HPV) infection; They are structurally different from other epithelial cells, in that, their nuclei are an irregular size, shape, or color
  • 141. Marc Imhotep Cray, MD Natural History of Squamous Intraepithelial Lesions (SILs) 141 Lesion Regress Persist Progress LSIL (CIN I) 60% 30% 10% (to HSIL) HSIL (CINII,III) 30% 60% 10% (to carcinoma)* *Progression within 10 years. LSIL, Low-grade SIL; HSIL, high-grade SIL Redrawn after Kumar V and Abbas AK. Robbins and Cotran Basis Pathology, 10th Ed. Philadelphia: Saunders, 2018, Fig. 19.1 , Pg. 718. Remember: Cervical Intraepithelial Neoplasia (CIN) Subtypes Grades I-III (or Squamous Intraepithelial Lesions Low-grade to High-grade )  CIN I (LSIL) is characterized by atypical undifferentiated cells only in lower third of epithelium, whereas,  CIN II (HSIL) superficial layer of cells still shows differentiation and koilocytosis  CIN III (HSIL) has atypia through > 2/3 thickness of epithelium, koilocytotic change usually is absent
  • 142. Marc Imhotep Cray, MD 142 Three Stages of squamous intraepithelial lesions (SIL) Spectrum of squamous intraepithelial lesions (SIL) with normal squamous epithelium for comparison: LSIL with koilocytotic atypia; HSIL w progressive atypia in all layers of the epithelium; and HSIL w diffuse atypia and loss of maturation (carcinoma in situ, far right image) Kumar V and Abbas AK. Robbins and Cotran Basis Pathology, 10th Ed. Philadelphia: Saunders, 2018, Fig. 19.6 , Pg. 719.
  • 143. Marc Imhotep Cray, MD 143 Cytologic features of squamous intraepithelial lesion (SIL) in a Papanicolaou Smear Superficial squamous cells may stain either red or blue. (A) Normal exfoliated superficial squamous epithelial cells. (B) Low-grade squamous intraepithelial lesion (LSIL). (C and D) Both high-grade squamous intraepithelial lesions (HSILs). Note the reduction in cytoplasm and the increase in the nucleus-to-cytoplasm ratio as the grade of the lesion increases. This observation reflects progressive loss of cellular differentiation on surface of cervical lesions from which these cells are exfoliated Kumar V and Abbas AK. Robbins and Cotran Basis Pathology, 10th Ed. Philadelphia: Saunders, 2018, Fig. 19.7 , Pg. 719.
  • 144. Marc Imhotep Cray, MD Dysplasia, Carcinoma in Situ, and Squamous Cell Carcinoma of Cervix (3) 144 Clinical Manifestations: ICC: Irregular vaginal bleeding; postcoital spotting; cervical ulceration; nonpurulent discharge; dysuria; invasive disease can obstruct ureters and lead to renal failure Treatment ICC: Hysterectomy; radiation therapy; prevention with HPV vaccine NB: Screening for cervical cancer with PAP smears should begin 3 years after intercourse and no later than age 21 and continue every 1–3 years until age 70
  • 145. Marc Imhotep Cray, MD 145 Normal cervix, gross Klatt EC. Robbins and Cotran Atlas of Pathology, 3rd Ed. 2015.
  • 146. Marc Imhotep Cray, MD 146 Normal cervix, microscopic Klatt EC. Robbins and Cotran Atlas of Pathology, 3rd Ed. 2015.
  • 147. Marc Imhotep Cray, MD 147 Normal cervical transformation zone, microscopic Klatt EC. Robbins and Cotran Atlas of Pathology, 3rd Ed. 2015.
  • 148. Marc Imhotep Cray, MD 148 Chronic cervicitis, gross Klatt EC. Robbins and Cotran Atlas of Pathology, 3rd Ed. 2015.
  • 149. Marc Imhotep Cray, MD 149 Chronic cervicitis, microscopic Klatt EC. Robbins and Cotran Atlas of Pathology, 3rd Ed. 2015.
  • 150. Marc Imhotep Cray, MD 150 Cervical squamous metaplasia, microscopic Klatt EC. Robbins and Cotran Atlas of Pathology, 3rd Ed. 2015.
  • 151. Marc Imhotep Cray, MD 151 Human papillomavirus effect, microscopic Klatt EC. Robbins and Cotran Atlas of Pathology, 3rd Ed. 2015.
  • 152. Marc Imhotep Cray, MD 152 Cervical squamous dysplasia, Pap smear Klatt EC. Robbins and Cotran Atlas of Pathology, 3rd Ed. 2015.
  • 153. Marc Imhotep Cray, MD 153 Cervical squamous carcinoma, Pap smear Klatt EC. Robbins and Cotran Atlas of Pathology, 3rd Ed. 2015.
  • 154. Marc Imhotep Cray, MD 154 Squamous cell carcinoma, gross Klatt EC. Robbins and Cotran Atlas of Pathology, 3rd Ed. 2015.
  • 155. Marc Imhotep Cray, MD 155 Squamous cell carcinoma, gross (2) Klatt EC. Robbins and Cotran Atlas of Pathology, 3rd Ed. 2015.
  • 156. Marc Imhotep Cray, MD 156 Squamous cell carcinoma, gross (3) Klatt EC. Robbins and Cotran Atlas of Pathology, 3rd Ed. 2015.
  • 157. Marc Imhotep Cray, MD Vignette #15 157 A 27-year-old woman presents to the emergency room with lower abdominal pain, chills, fever, and a purulent vaginal discharge. After taking a history, you learn that she has had multiple sexual partners over the last year and has not practiced safe sex. She states that the pain started 3 days ago with subsequent fevers, chills, and night sweats. On physical examination, she is febrile with a temperature of 102.4°F. Pelvic examination is significant for cervical motion tenderness with foul-smelling, purulent, cervical discharge. Her pregnancy test is negative. You admit the patient to the hospital and you begin her on IV cefoxitin plus doxycycline while awaiting the results of her endocervical culture. What is the Diagnosis?
  • 158. Marc Imhotep Cray, MD Pelvic Inflammatory Disease 158 Etiology and Epidemiology: Common causes include Chlamydia trachomatis (subacute), Neisseria gonorrhoeae (acute), Gardnerella vaginalis, and Trichomonas vaginalis; Usually occurs in young, nulliparous, sexually active women with multiple partners Pathology: Fallopian tubes: Edematous tubal serosa with fibrin covering; purulent exudate in lumen; collections of pus may form a pyosalpinx or degrading pus may form a hydrosalpinx (water-filled fallopian tubes) Infection can also involve the ovaries and other pelvic structures
  • 159. Marc Imhotep Cray, MD PID cont. 159 Clinical Manifestations: High fever; lower abdominal pain; cervical motion tenderness (chandelier sign); purulent cervical discharge; RUQ pain indicates perihepatitis (Fitz-Hugh-Curtis syndrome); Salpingitis is a risk factor for ectopic pregnancy, infertility, chronic pelvic pain, and adhesions Treatment: Antibiotics effective against causative organism Note: Ectopic pregnancy occurs most often in fallopian tubes, but may also occur in ovary, abdominal cavity, or cervix  Risk factors include salpingitis, endometriosis, and tubal ligation  Clinical manifestations include severe abdominal pain 6 weeks after LMP and elevated hCG which is lower than normal for pregnancy stage
  • 160. Marc Imhotep Cray, MD Pathogenesis of gonococcal infections 160  Neisseria gonorrhoeae is a gram-negative diplococcus whose surface pili form a barrier against phagocytosis by neutrophils  Pili contain an immunoglobulin A (IgA) protease that digests IgA on luminal surface of mucous membranes of urethra, endocervix and fallopian tube thereby facilitating attachment of gonococci  Gonococci cause endocervicitis, vaginitis and salpingitis  In men, gonococci attached to mucous membrane of urethra cause urethritis and, sometimes, urethral stricture  GC may also attach to sperm heads and be carried into the fallopian tube Penetration of mucous membrane by gonococci leads to stricture of fallopian tube, pelvic inflammatory disease (PID) or tuboovarian abscess Rubin R and Strayer DS Eds. Rubin’s Pathology: Clinicopathologic Foundations of Medicine, 6th Ed. Baltimore: Lippincott Williams & Wilkins, 2012; Fig. 9.18, 394. Click for large view
  • 161. Marc Imhotep Cray, MD 161 Gonorrhea of the fallopian tube Cross-section of a “pus tube” shows thickening of wall and a lumen swollen with pus Rubin R and Strayer DS Eds. Rubin’s Pathology: Clinicopathologic Foundations of Medicine, 6th Ed. Baltimore: Lippincott Williams & Wilkins, 2012; Fig. 9.19, 394.
  • 162. Marc Imhotep Cray, MD 162 Acute salpingitis, microscopic Klatt EC. Robbins and Cotran Atlas of Pathology, 3rd Ed. 2015.
  • 163. Marc Imhotep Cray, MD 163 Tubo-ovarian abscess, gross Klatt EC. Robbins and Cotran Atlas of Pathology, 3rd Ed. 2015.
  • 164. Marc Imhotep Cray, MD 164 Ectopic pregnancy, gross Klatt EC. Robbins and Cotran Atlas of Pathology, 3rd Ed. 2015.
  • 165. Marc Imhotep Cray, MD Vignette #16 165 After vaginally delivering a newborn from a 32-year-old woman, you find the mother to be hemorrhaging an abnormally large amount of blood. You recall that she has had one previous C-section and extensive scarring. Given the amount of blood after delivery and the abnormal gross appearance of the placental remnants, you believe that the patient’s pregnancy was complicated by a defective decidual layer that allowed the placenta to attach directly to the myometrium. You call the operating room to set up for an emergency hysterectomy, which you believe may be necessary to control the patient’s massive hemorrhage. What is the Diagnosis?
  • 166. Marc Imhotep Cray, MD Placental Attachment Abnormalities (Abruptio Placentae, Placenta Previa, Placenta Accreta) 166 Etiology: Abruptio placentae: May be associated with DIC; increased risk with smoking, cocaine, and hypertension Placenta accreta: Predisposed by prior C-section scars or endometrial inflammation Placenta previa: Predisposed by prior C-section scars Pathophysiology: Abruptio placentae: Premature separation of placenta Placenta accreta: Defective decidual layer allows placenta to attach directly to myometrium Placenta previa: Attachment of placenta to lower uterine segment or cervix; may occlude cervical os; may coexist with placenta accreta
  • 167. Marc Imhotep Cray, MD Placental Attachment Abnormalities cont. 167 Clinical Manifestations: Abruptio placentae: Painful uterine bleeding usually during third trimester; can result in fetal death Placenta accreta: Massive hemorrhage after delivery Placenta previa: Painless bleeding in any trimester; premature labor Treatment: Abruptio placentae: Immediate fetal delivery; control of maternal bleeding Placenta accreta: Hysterectomy may be necessary to stop bleeding Placenta previa: Bed rest; possible hospitalization
  • 168. Marc Imhotep Cray, MD Vignette #17 168 A 31-year-old pregnant woman is brought to the emergency room complaining of headaches and blurred vision of 1-week duration. She is currently 32 weeks into her first pregnancy. On physical examination, you find edema of the face and lower extremities. Her blood pressure is 160/100, but she has no prior history of hypertension. Laboratory studies show a mild thrombocytopenia, elevated AST and ALT, and significant proteinuria. Based on these findings, you admit the patient for immediate bed rest, close monitoring, and blood pressure control. You tell the patient that it is possible that you may need to deliver her baby early in order to treat the patient’s current condition. What is the Diagnosis?
  • 169. Marc Imhotep Cray, MD Preeclampsia and Eclampsia 169 Etiology and Epidemiology: Risk factors include preexisting hypertension, diabetes, chronic renal disease, autoimmune disorders, or twin gestation Affects 7% of pregnant women usually during the third trimester of a woman’s first pregnancy Pathology and Pathophysiology: Placenta: Evidence of infarct; presence of retroplacental hematomas; decreased vascularity; fibrinoid necrosis  Pathophysiology: Intrinsic defect in invading cytotrophoblast leads to remodeling of uterine vasculature and resulting placental ischemia; decreased placental perfusion induces vasoconstrictive effects leading to toxemic hypertension in susceptible women
  • 170. Marc Imhotep Cray, MD Preeclampsia and Eclampsia cont. 170 Clinical Manifestations: Preeclampsia: Triad of hypertension, proteinuria, and edema; associated Sx include headache, blurry vision, and abdominal pain Eclampsia: Preeclampsia triad plus seizures, altered mentation, hyperreflexia, and possibly DIC Lab findings: Thrombocytopenia, elevated LFTs, hyperuricemia, hemolytic anemia Treatment: Preeclampsia: Delivery of fetus as soon as possible; bed rest; salt restriction; treatment of hypertension Eclampsia: Medical emergency; treat with IV magnesium sulfate and diazepam; delivery of fetus NB: HELLP syndrome refers to hemolysis, elevated LFTs, and low platelets and is a severe form of preeclampsia (Video on HELLP syndrome & “USMLE”)
  • 171. Marc Imhotep Cray, MD THE END See next slide for further study. 171
  • 172. Marc Imhotep Cray, MD Companion Tools and Resources (online) 172 Discipline (Pathology) Track Folder  Reproductive Pathology Organ Systems Track Folders  M and FM Reproductive System & FM Breast  Pregnancy, Childbirth, & the Puerperium Textbooks: Kumar V and Abbas AK. Robbins and Cotran Basis Pathology, 10th Ed. Philadelphia: Saunders, 2018. Rubin R and Strayer DS Eds. Rubin’s Essential of Pathology:, 6th Ed. Baltimore: Lippincott Williams & Wilkins, 2014.